Download as pdf or txt
Download as pdf or txt
You are on page 1of 78

Serous Fluid Cytopathology

2
Qing Kay Li and Walid E. Khalbuss

Contents
2.1 Image-Based Questions 1–50 (50 Images)........................ 124
2.2 Text-Based Questions 51–100 ............................................ 174
2.3 Answers and Discussion of Image-Based
Questions 1–50 .................................................................... 179
2.4 Answers and Discussion of Text-Based
Questions 51–100 ................................................................ 188
Reading List................................................................................. 197

Q.K. Li, MD, Ph.D. (*)


Department of Pathology, The Johns Hopkins University School
of Medicine, The Johns Hopkins Bayview Medical Center,
4940 Eastern Avenue, AA Building, Room 154B, Baltimore,
MD 21224-2780, USA
e-mail: qli23@jhmi.edu
W.E. Khalbuss, MD, PhD, FIAC.
Department of Pathology, GE Clarient Diagnostic Services,
31 Columbia, Aliso Viejo, California, 92656, USA
e-mail: walid.khalbuss@ge.com

W.E. Khalbuss, Q.K. Li (eds.), Diagnostic Cytopathology Board Review and Self-Assessment, 121
DOI 10.1007/978-1-4939-1477-7_2, © Springer Science+Business Media New York 2015
122 Q.K. Li and W.E. Khalbuss

Table 2.1 Benign components in serous fluid


Descriptions Differentials
Mesothelial cells Sheets and/or dispersed individual cells with round nuclei, small Organized cellular arrangements and/or dispersed
nucleoli, granular chromatin, dense cytoplasm and clear outer rim,individual cells. Nuclei may show mild atypia, but
and “windows” between cells with smooth nuclear membrane; no hyperchromasia
DD: well-differentiated adenocarcinoma
Reactive mesothelial Small clusters of cells (less than10–20 cells) or dispersed A spectrum of changes ranges from normal to
cells individual cells with enlarged nuclei, coarse chromatin, prominent markedly atypical cells. Binucleation and
nucleoli, dense cytoplasm and clear outer rim, and “windows” multinucleation are common
between cells DD: well-differentiated adenocarcinoma
Histiocytes Loosely formed two-dimensional clusters or dispersed individual Cells have coffee bean-shaped nuclei, cytoplasmic
cells, with coffee bean-shaped nuclei, fine chromatin, vacuoles and normal N/C ratios
inconspicuous nucleoli, and foamy cytoplasm DD: well-differentiated adenocarcinoma
Eosinophils Granulocytes with brick-red cytoplasmic granules after staining Cells with eosinophilic intracytoplasmic granules
with eosin (a red dye) using the Papanicolaou stain method DD: neutrophils
Lupus erythematosus Neutrophils or macrophages with intracytoplasmic hematoxylin Cells with eccentric nuclei and large homogeneous
(LE) cells bodies intracytoplasmic bodies (hematoxylin bodies)
DD: signet ring cells
Hematoxylin body Homogeneous and glassy intracytoplasmic body (denatured Presence in the cytoplasm of neutrophil or
nuclear material), green or blue or purple in color with the macrophage (LE cells), commonly seen in lupus
Papanicolaou stain, and magenta in color with the Diff-Quik stain pleuritis
Psammoma body Round collections of calcium which is covered with a thin layer Calcium in the center of the spheres. It may be
of benign mesothelial cells. It stains purple in color with the found in both benign and malignant conditions
Papanicolaou method DD: collagen ball
Magenta body Variably sized red-to-purple perinuclear inclusions seen by the Seen in metastatic breast carcinomas
Diff-Quik stains
Collagen ball Spheres of collagen which is covered with a thin layer of benign Nonspecific finding
mesothelial cells DD: psammoma body. Metastatic mucinous
adenocarcinomas, papillary serous carcinomas of
the ovary, and mesotheliomas
Charcot–Leyden Needle-shaped orangeophilic color crystal, a by-product of Associated with allergy
crystal eosinophil degranulation DD: calcium oxalate crystals
2 Serous Fluid Cytopathology 123

Table 2.2 Malignant conditions in serous fluid


Conditions Descriptions Differentials
Mesothelioma Numerous three-dimensional clusters (with more Cellular clusters with scallop border and cytoplasmic vacuoles.
than 20–30 tumor cells). Tumor cells have large Tumor cells may maintain low N/C ratios and “windows”
nuclei, coarse chromatin, and irregular nuclear between cells. Cytoplasm is dense with “hyaluronic acid mucin”
membrane DD: adenocarcinoma
Adenocarcinoma Three-dimensional cell clusters with smooth border Columnar cells have high N/C ratios with irregular nuclear
and/or acinar/papillae arrangements of columnar membrane, coarse chromatin, prominent nucleoli, and
cells. Tumor cells have high N/C ratios, prominent cytoplasmic vacuole (mucin production). No “window” between
nucleoli, and “lacy” cytoplasm with vacuolization cells
DD: mesothelioma, poorly differentiated nonkeratinizing
squamous cell carcinoma
Squamous cell Discohesive and scattered individual polymorphic Polygonal, rounded, elongated, or tadpole-shaped cells with
carcinoma cells or loosely formed two-dimensional cell clusters large pyknotic nuclei, smudgy chromatin and dense cytoplasm.
with or without keratinization No “window” between cells
DD: poorly differentiated adenocarcinoma
Small cell carcinoma Tight clusters of small hyperchromatic cells (two to Fine chromatin (salt-and-pepper appearance), mitoses, necrosis,
three times the size of mature lymphocytes) with and apoptotic bodies
nuclear molding and crowding, nuclear stripes DD: lymphoma, basaloid squamous cell carcinoma, and poorly
(breakdown of nuclear material), inconspicuous differentiated adenocarcinoma
nuclei, scant cytoplasm
Undifferentiated Loosely cohesive clusters, syncytial sheets or Malignant cells are huge with large nucleoli. Numerous mitoses
large cell carcinoma scattered polymorphic large cells with high N/C DD: poorly differentiated adenocarcinoma and sarcomas
ratios, single or multiple prominent nucleoli and
feathery cytoplasm
Melanoma Scattered individual large cells with prominent Discohesive malignant cells with prominent nucleoli
nucleoli and cytoplasmic melanin pigments. DD: poorly differentiated carcinoma, Hodgkin lymphoma
Binucleation with “mirror” arrangement.
Pseudointranuclear inclusions
Lymphoma Dispersed individual atypical lymphoid cells with The size of tumor cells ranges from small to large depending on
(non-Hodgkin clumped chromatin and irregular nuclear membrane. the type of lymphoma. Monomorphous population of
lymphoma) Increased mitotic activity. Lymphoglandular bodies lymphocytes is a characteristic of SLL/CLL
in the background of the slide DD: reactive lymphocytes, small cell carcinoma, poorly
differentiated carcinoma
Non-epithelial cell Individual or clusters of spindle cells, small round DD: muscle, nerve tumors, small round cell tumor, and sarcomas
neoplasma cells or pleomorphic tumor cells
124 Q.K. Li and W.E. Khalbuss

2.1 Image-Based Questions 1–50


(50 Images)

Fig. 2.1

Q-1. What is the diagnosis of this peritoneal washing


specimen?
(a) Metastatic adenocarcinoma
(b) Serous carcinoma of ovary
(c) Collagen ball
(d) Mesothelioma
2 Serous Fluid Cytopathology 125

Fig. 2.2

Q-2. What is the diagnosis of cells seen in the pleural


effusion from a patient with systemic lupus
erythematosus?
(a) Lupus erythematosus (LE) cells
(b) Tart cells
(c) Reactive mesothelial cells
(d) Mucin-producing cells
126 Q.K. Li and W.E. Khalbuss

Fig. 2.3

Q-3. What is the diagnosis of cells seen in this pericardial


effusion from a 52-year-old female patient with a
history of breast carcinoma and melanoma?
(a) Metastatic lung adenocarcinoma
(b) Reactive mesothelial cells
(c) Metastatic melanoma
(d) Metastatic breast lobular carcinoma
2 Serous Fluid Cytopathology 127

Fig. 2.4

Q-4. What is the diagnosis of cells seen in this ascites from


a 24-year-old male with a retroperitoneal mass?
(a) Small cell carcinoma
(b) Lymphoma
(c) Ewing’s sarcoma
(d) Metastatic adenocarcinoma
128 Q.K. Li and W.E. Khalbuss

Fig. 2.5

Q-5. What is the diagnosis of cells seen in this pleural


effusion from a 78-year-old female patient with
pleural thickening?
(a) Metastatic lung adenocarcinoma
(b) Reactive mesothelial cells
(c) Metastatic melanoma
(d) Mesothelioma
2 Serous Fluid Cytopathology 129

Fig. 2.6

Q-6. What is the diagnosis of cells seen in this ascites


from an 8-year-old boy with a retroperitoneal mass?
(a) Metastatic adenocarcinoma
(b) Reactive mesothelial cells
(c) Metastatic melanoma
(d) Metastatic rhabdomyosarcoma
130 Q.K. Li and W.E. Khalbuss

Fig. 2.7

Q-7. An 83-year-old male patient with lymphadenopathy


developed pleural effusions. The specimen consists
predominately of a monomorphous population of
lymphocytes with apoptosis. Which one the following
diagnoses is correct?
(a) Metastatic poorly differentiated adenocarcinoma
(b) Small cell carcinoma
(c) Metastatic melanoma
(d) Lymphomatous effusion most consistent with
malignant lymphoma
2 Serous Fluid Cytopathology 131

Fig. 2.8

Q-8. A 77-year-old male smoker with a right upper lung


mass developed a pleural effusion. Which one of the
following diagnoses is correct?
(a) Metastatic poorly differentiated adenocarcinoma
(b) Metastatic small cell lung carcinoma
(c) Metastatic melanoma
(d) Lymphoma
132 Q.K. Li and W.E. Khalbuss

Fig. 2.9

Q-9. In this ascites from a 58-year-old female, the


cytological findings are shown in this photo. What is
the correct diagnosis?
(a) Metastatic ovarian serous carcinoma
(b) Mesothelial cell hyperplasia
(c) Endosalpingiosis
(d) Metastatic adenocarcinoma
2 Serous Fluid Cytopathology 133

Fig. 2.10

Q-10. A 64-year-old female smoker with a lung mass


developed a pleural effusion. On cytospin prepara-
tion, the smear consists of predominantly small- to
intermediate-sized lymphocytic cells with evenly
distributed chromatin. Which one of the following
diagnoses is correct?
(a) Metastatic poorly differentiated adenocarcinoma
(b) Metastatic small cell lung carcinoma
(c) Lymphoma
(d) Lymphocytosis (lymphocytic effusion)
134 Q.K. Li and W.E. Khalbuss

Fig. 2.11

Q-11. A 34-year-old female with a history of melanoma


now develops a lung mass and pleural effusion.
Which one the following diagnoses is correct?
(a) Metastatic poorly differentiated adenocarcinoma
of the lung
(b) Metastatic small cell lung carcinoma
(c) Metastatic melanoma
(d) Lymphoma
2 Serous Fluid Cytopathology 135

Fig. 2.12

Q-12. What is the diagnosis of cells seen in this pleural


effusion?
(a) Metastatic lung adenocarcinoma
(b) Reactive mesothelial cells
(c) Metastatic melanoma
(d) Mesothelioma
136 Q.K. Li and W.E. Khalbuss

Fig. 2.13

Q-13. A 48-year-old female with a history of melanoma


now develops a pleural effusion. Which one of the
following diagnoses is correct?
(a) Metastatic renal cell carcinoma
(b) Metastatic small cell lung carcinoma
(c) Metastatic melanoma
(d) Lymphoma
2 Serous Fluid Cytopathology 137

Fig. 2.14

Q-14. A 68-year-old male with a head and neck mass now


develops a pleural effusion. Which one of the follow-
ing diagnoses is correct?
(a) Metastatic squamous cell carcinoma
(b) Metastatic poorly differentiated adenocarcinoma
(c) Metastatic melanoma
(d) Lymphoma
138 Q.K. Li and W.E. Khalbuss

Fig. 2.15

Q-15. A 48-year-old female with a history of melanoma


now develops a lung mass and pleural effusion.
What is the diagnosis of cells seen in this pleural
effusion?
(a) Metastatic lung adenocarcinoma
(b) Reactive mesothelial cells
(c) Metastatic melanoma
(d) Mesothelioma
2 Serous Fluid Cytopathology 139

Fig. 2.16

Q-16. A 62-year-old female with a history of “carcinoma”


now develops a pleural effusion. What is the diagnosis
of cells seen in this pleural effusion?
(a) Metastatic renal cell carcinoma
(b) Reactive mesothelial cells
(c) Metastatic melanoma
(d) Metastatic signet ring cell carcinoma
140 Q.K. Li and W.E. Khalbuss

Fig. 2.17

Q-17. A 70-year-old female with a history of right “kidney


tumor” develops a right-side pleural effusion. What is
the diagnosis of cells seen in this pleural effusion?
(a) Metastatic papillary renal cell carcinoma
(b) Reactive mesothelial cells
(c) Mesothelioma
(d) Metastatic poorly differentiated adenocarcinoma
of the lung
2 Serous Fluid Cytopathology 141

Fig. 2.18

Q-18. A 61-year-old male with a clinical presentation of a


lung mass develops a left pleural effusion. What is
the diagnosis of cells seen in this pleural effusion?
(a) Metastatic adenocarcinoma of the lung
(b) Reactive mesothelial cells
(c) Mesothelioma
(d) Metastatic poorly differentiated small cell
carcinoma
142 Q.K. Li and W.E. Khalbuss

Fig. 2.19

Q-19. Which one of the following immunomarkers has a


better sensitivity for this metastatic carcinoma of the
lung?
(a) TTF1 (thyroid transcription factor 1)
(b) Synaptophysin
(c) Chromogranin A
(d) CD56
2 Serous Fluid Cytopathology 143

Fig. 2.20

Q-20. A 2-year-old female with an abdominal mass devel-


oped ascites. On surgical resection specimen, Homer–
Wright rosettes are identified. The ascites cytospin
was shown here. Which panel of following immuno-
markers can help you confirm the diagnosis?
(a) Cytokeratin and TTF1
(b) Neuron-specific enolase (NSE), synaptophysin,
S100, and glial fibrillary acidic protein (GFAP)
(c) Chromogranin A, synaptophysin, and CD56
(d) CD20, CD3, and CD56
144 Q.K. Li and W.E. Khalbuss

Fig. 2.21

Q-21. A 70-year-old male with a liver mass and lung


masses develops a left pleural effusion. What is the
diagnosis of cells seen in this pleural effusion?
(a) Metastatic poorly differentiated adenocarcinoma
of the lung
(b) Reactive mesothelial cells
(c) Metastatic hepatocellular carcinoma
(d) Metastatic poorly differentiated squamous cell
carcinoma
2 Serous Fluid Cytopathology 145

Fig. 2.22

Q-22. A 72-year-old female with a history of colon cancer


now develops a lung mass and pleural effusion.
What is the diagnosis of cells seen in this pleural
effusion?
(a) Metastatic carcinoma of the breast
(b) Reactive mesothelial cells
(c) Metastatic colonic adenocarcinoma
(d) Mesothelioma
146 Q.K. Li and W.E. Khalbuss

Fig. 2.23

Q-23. A 69-year-old male smoker presents with a right lung


mass and pleural effusion. What is the diagnosis of
cells seen in this pleural effusion?
(a) Metastatic papillary adenocarcinoma of the lung
(b) Reactive mesothelial cells
(c) Metastatic colonic adenocarcinoma
(d) Metastatic mucinous adenocarcinoma of the lung
2 Serous Fluid Cytopathology 147

Fig. 2.24

Q-24. What is the diagnosis of cells seen in this pleural


effusion?
(a) Metastatic adenocarcinoma
(b) Reactive mesothelial cells
(c) Mesothelioma
(d) Metastatic squamous cell carcinoma
148 Q.K. Li and W.E. Khalbuss

Fig. 2.25

Q-25. An HIV-positive patient develops bilateral pleural


effusions. What is the diagnosis of cells seen in this
pleural effusion?
(a) Metastatic adenocarcinoma
(b) Primary effusion lymphoma
(c) Reactive mesothelial cells
(d) Metastatic squamous cell carcinoma
2 Serous Fluid Cytopathology 149

Fig. 2.26

Q-26. A 79-year-old patient with neck lymphadenopathy


and mediastinal mass develops bilateral pleural
effusions. What is the diagnosis of cells seen in this
pleural effusion?
(a) Metastatic adenocarcinoma
(b) SLL/CLL
(c) Reactive mesothelial cells
(d) Hodgkin lymphoma
150 Q.K. Li and W.E. Khalbuss

Fig. 2.27

Q-27. A 47-year-old male with history of hepatitis C and


numerous liver nodules develops pleural effusion
and ascites. What is the diagnosis of cells seen in
this pleural effusion?
(a) Metastatic signet ring cell carcinoma
(b) SLL/CLL
(c) Reactive mesothelial cells
(d) Hodgkin lymphoma
2 Serous Fluid Cytopathology 151

Fig. 2.28

Q-28. A 58-year-old male smoker with history of “lung


cancer” develops a pleural effusion. What is the diag-
nosis of cells seen in this pleural effusion?
(a) Metastatic adenocarcinoma
(b) Metastatic squamous cell carcinoma
(c) Reactive mesothelial cells
(d) Metastatic melanoma
152 Q.K. Li and W.E. Khalbuss

Fig. 2.29

Q-29. Surgical peritoneal washing was performed for staging


purpose. What is the diagnosis of this cytological
specimen?
(a) Metastatic adenocarcinoma of the colon
(b) Metastatic squamous cell carcinoma
(c) Reactive mesothelial cells
(d) Metastatic serous carcinoma
2 Serous Fluid Cytopathology 153

Fig. 2.30

Q-30. A 58-year-old female with history of breast cancer


and radiation and chemotherapy develops a pleural
effusion. What is the diagnosis of cells seen in this
pleural effusion?
(a) Metastatic adenocarcinoma
(b) Metastatic squamous cell carcinoma
(c) Reactive mesothelial cells
(d) Metastatic melanoma
154 Q.K. Li and W.E. Khalbuss

Fig. 2.31

Q-31. The diagnosis of a malignant pleural effusion was


made in this specimen. Which panel of immuno-
markers can help you confirm the diagnosis?
(a) Cytokeratin and CK7
(b) Calretinin, TTF, and BerEP4
(c) Chromogranin A, synaptophysin, and CD56
(d) CD20, CD3, and CD56
2 Serous Fluid Cytopathology 155

Fig. 2.32

Q-32. The diagnosis of a malignant pleural effusion was


made on this specimen. What is the diagnosis of
cells seen in this pleural effusion?
(a) Metastatic mucinous adenocarcinoma
(b) Metastatic squamous cell carcinoma
(c) Reactive mesothelial cells
(d) Metastatic small cell carcinoma
156 Q.K. Li and W.E. Khalbuss

Fig. 2.33

Q-33. Surgical peritoneal washing was performed. What is


the diagnosis of this cytological specimen (cell
block preparation)?
(a) Metastatic adenocarcinoma of the colon
(b) Mesothelioma
(c) Reactive mesothelial cells
(d) Metastatic serous carcinoma of the ovary
2 Serous Fluid Cytopathology 157

Fig. 2.34

Q-34. A 22-year-old female with an ovarian mass. During


her surgery, pelvic washing was performed. What is
the diagnosis of this cytological specimen?
(a) Metastatic adenocarcinoma of the colon
(b) Metastatic dysgerminoma
(c) Reactive mesothelial cells
(d) Metastatic serous carcinoma of the ovary
158 Q.K. Li and W.E. Khalbuss

Fig. 2.35

Q-35. Surgical peritoneal washing was performed. What is


the diagnosis of this cytological specimen?
(a) Metastatic adenocarcinoma of the colon
(b) Mesothelioma
(c) Reactive mesothelial cells
(d) Metastatic serous carcinoma of the ovary
2 Serous Fluid Cytopathology 159

Fig. 2.36

Q-36. Surgical peritoneal washing was performed. What is


the diagnosis of this cytological specimen?
(a) Metastatic adenocarcinoma of the colon
(b) Mesothelioma
(c) Reactive mesothelial cells
(d) Metastatic serous carcinoma of the ovary
160 Q.K. Li and W.E. Khalbuss

Fig. 2.37

Q-37. Surgical peritoneal washing was performed. What is


the diagnosis of this cytological specimen?
(a) Metastatic adenocarcinoma of the colon
(b) Reactive mesothelial cells
(c) Mesothelioma
(d) Metastatic serous carcinoma of the ovary
2 Serous Fluid Cytopathology 161

Fig. 2.38

Q-38. A 35-year-old with an ovarian mass had a surgical


resection of the lesion. Pelvic washing was per-
formed during the procedure. The resected lesion
reveals a hemorrhagic cyst. What is the diagnosis of
this pelvic washing specimen?
(a) Metastatic adenocarcinoma of the colon
(b) Reactive mesothelial cells
(c) Endometriosis
(d) Metastatic serous carcinoma of the ovary
162 Q.K. Li and W.E. Khalbuss

Fig. 2.39

Q-39. Surgical peritoneal washing was performed. What is


the diagnosis of this cytological specimen?
(a) Metastatic adenocarcinoma
(b) Reactive mesothelial cells
(c) Mesothelioma
(d) Metastatic serous carcinoma of the ovary
2 Serous Fluid Cytopathology 163

Fig. 2.40

Q-40. An 87-year-old female nonsmoker with a lung mass


develops a pleural effusion. What is the diagnosis of
this cytological specimen?
(a) Metastatic adenocarcinoma of the lung
(b) Reactive mesothelial cells
(c) Metastatic melanoma
(d) Metastatic squamous cell carcinoma
164 Q.K. Li and W.E. Khalbuss

Fig. 2.41

Q-41. What is the diagnosis of this pleural effusion


specimen?
(a) Metastatic adenocarcinoma
(b) Reactive mesothelial cells
(c) Mesothelioma
(d) Metastatic serous carcinoma of the ovary
2 Serous Fluid Cytopathology 165

Fig. 2.42

Q-42. Which one of the following diagnoses is correct?


(a) Metastatic poorly differentiated adenocarcinoma
of the lung
(b) Metastatic small cell lung carcinoma
(c) Metastatic melanoma
(d) Lymphoma
166 Q.K. Li and W.E. Khalbuss

Fig. 2.43

Q-43. Pleural effusion was found in a patient with rheuma-


toid arthritis. What is the diagnosis of this pleural
effusion?
(a) Rheumatoid pleuritis
(b) Reactive mesothelial cells
(c) Lymphocytosis
(d) Lymphoma
2 Serous Fluid Cytopathology 167

Fig. 2.44

Q-44. What is the diagnosis of this cytological specimen?


(a) Metastatic adenocarcinoma
(b) Mesothelioma
(c) Reactive mesothelial cells
(d) Metastatic serous carcinoma of the ovary
168 Q.K. Li and W.E. Khalbuss

Fig. 2.45

Q-45. What is the diagnosis of this pleural effusion


specimen?
(a) Metastatic adenocarcinoma
(b) Mesothelioma
(c) Reactive mesothelial cells
(d) Metastatic squamous cell carcinoma
2 Serous Fluid Cytopathology 169

Fig. 2.46

Q-46. A paracentesis was performed in a 46-year-old


female patient with abdominal pain and distension.
What is the diagnosis of cytological findings?
(a) Metastatic adenocarcinoma
(b) Mesothelioma
(c) Reactive mesothelial cells
(d) Pseudomyxoma peritonei
170 Q.K. Li and W.E. Khalbuss

Fig. 2.47

Q-47. What is the diagnosis of this pleural effusion


specimen?
(a) Metastatic adenocarcinoma
(b) Reactive mesothelial cells
(c) Mesothelioma
(d) Metastatic serous carcinoma of the ovary
2 Serous Fluid Cytopathology 171

Fig. 2.48

Q-48. In this ascites specimen, what is the correct diagnosis?


(a) Metastatic serous carcinoma
(b) Mesothelial cell hyperplasia
(c) Endosalpingiosis
(d) Metastatic squamous cell carcinoma
172 Q.K. Li and W.E. Khalbuss

Fig. 2.49

Q-49. What is the diagnosis of this pleural effusion from a


66-year-old male with HIV infection?
(a) Diffuse large B cell lymphoma
(b) Hodgkin lymphoma
(c) Burkitt lymphoma
(d) Lymphocytic pleural effusion
2 Serous Fluid Cytopathology 173

Fig. 2.50

Q-50. What is the correct diagnosis of this pleural effusion?


(a) Diffuse large B cell lymphoma
(b) Hodgkin lymphoma
(c) Eosinophilic pleural effusion
(d) Lymphocytic pleural effusion
174 Q.K. Li and W.E. Khalbuss

2.2 Text-Based Questions 51–100 Q-56. Radiation- and chemotherapy-induced mesothelial


changes include all of the following, except:
Q-51. In a pleural effusion, all of the following features are (a) Cytomegaly with proportionate nuclear
seen in reactive mesothelial cells, except: enlargement
(a) Scattered individual “atypical” mesothelial cells (b) Multinucleation and a vesicular chromatin
(b) A spectrum of cellular changes, ranging from pattern
“normal” to “atypical” mesothelial cells (c) Prominent nucleoli
(c) Variation in nuclear size, binucleation, and (d) Hyperchromatic nucleoli and clumped
multinucleation chromatin
(d) Numerous large three-dimensional cellular clusters
Q-57. A 60-year-old male patient presents with left pleu-
Q-52. Which panel of immunohistochemical markers is ral effusion. An ultrasound-guided thoracentesis
the most useful in the differential diagnosis of meso- was performed. On cytology smear and cell block
thelioma from a metastatic adenocarcinoma of the section, lymphoid population and rare mesothelial
lung? cells were identified. Which of the following fea-
(a) AE1/AE3, CK7, TTF1, calretinin, Ki67, WT1, tures is considered an “abnormal” lymphoid
and P53 population?
(b) BerEP4, Napsin A, TTF1, calretinin, WT1, and (a) Polymorphous population of lymphocytes
P53 (b) “Lacunar” space on cell block section
(c) AE1/AE3, CK7, CK20, TTF1, calretinin, and (c) Monomorphous population of lymphocytes
Ki67 (d) Lymphoglandular bodies
(d) BerEP4, Napsin A, TTF1, calretinin, and Ki67
Q-58. On a scant effusion specimen, which of the follow-
Q-53. Which one of the following is the most useful feature ing tumors is most likely to be mistaken for “reac-
for separating reactive mesothelial cells (reactive tive mesothelial cells”?
atypia) from a well-differentiated lung (a) Adenocarcinoma with lepidic growth pattern
adenocarcinoma? (bronchoalveolar carcinoma)
(a) Two- or three-dimensional clusters of cells with (b) Poorly differentiated squamous cell carcinoma
nuclear variations (c) Basaloid carcinoma
(b) “Window” between atypical cells (d) Small cell carcinoma
(c) Normal to low N/C ratio
(d) Binucleation and/or multinucleation Q-59. The differential diagnosis of eosinophils in pleural
effusion (eosinophilic pleural effusion) includes all
Q-54. An ultrasound-guided thoracentesis was performed of the following, except:
on a 65-year-old male smoker who has a right pleu- (a) Repeat tap (thoracentesis)
ral effusion and a right lower pleura-based mass. (b) Renal transplantation
The cytospin smear reveals numerous large clusters (c) Mesothelioma
of epithelioid cells with well-defined cell borders, (d) Tuberculosis
large nuclei and prominent nucleoli, and intercellu-
lar “windows.” What is the most likely diagnosis? Q-60. Fluid cytology is more sensitive than a “blind”
(a) Reactive mesothelial cells serosal biopsy for the detection of serosal
(b) Well-differentiated adenocarcinoma of the lung malignancy.
(c) Poorly differentiated carcinoma (a) True
(d) Mesothelioma (b) False

Q-55. All of the following features can be seen in a squa- Q-11. The most common cause of eosinophilic pleural
mous cell carcinoma, except: effusion is:
(a) Smudgy chromatin (a) Blood in the pleural space (hemothorax)
(b) Dense cytoplasm (b) Lung cancers
(c) Prominent nucleoli (c) Air in the pleural space (pneumothorax)
(d) Shared cell borders (d) Tuberculosis
2 Serous Fluid Cytopathology 175

Q-62. Which of the following irritation/trauma induces a (a) Colloidal iron


rapid development of eosinophilic pleural effusion (b) Alcian blue
in hours? (c) PAS (Periodic acid–Schiff)
(a) Blood in the pleural space (hemothorax) (d) Mucicarmine
(b) Air in the pleural space (pneumothorax)
(c) Pleural biopsy Q-69. A cytological smear of ascites reveals numerous
(d) Repeat thoracentesis two-dimensional clusters and/or sheets of spindle
cells; the differential diagnosis of the lesion includes
Q-63. The most common cause of eosinophilic ascites is: all of the following, except:
(a) Metastatic colon cancers (a) Spindle cell carcinoma
(b) Metastatic lung cancers (b) Sarcomatoid mesothelioma
(c) Blood in the peritoneal space (c) Metastatic melanoma
(d) Cirrhosis (d) Poorly differentiated adenocarcinoma

Q-64. Metastatic squamous cell carcinomas in the fluid Q-70. Which feature is not seen in a small cell
cytology can be distinguished from metastatic ade- carcinoma?
nocarcinomas based on the presence of the follow- (a) Paranuclear blue bodies in the cytoplasm
ing features, except: (b) The size of tumor cells are two- to threefold of
(a) Keratinization mature lymphocytes
(b) Mucin production (cytoplasmic mucin) (c) Abundant cytoplasm
(c) Pseudoacinar arrangement of tumor cells (d) Nuclear crowding and molding
(d) Prominent nucleoli
Q-71. Which of the following cytological features is help-
Q-65. A thoracentesis was performed on a 65-year-old ful in the differential diagnosis of large cell neuroen-
male smoker who presented with a right upper lobe docrine tumor from a poorly differentiated
infiltration and pleural effusion. On the cytological adenocarcinoma in a pleural effusion specimen?
preparation, large sheets and tight clusters of atypi- (a) Fine (salt-and-pepper) chromatin pattern
cal epithelioid cells are identified; some of them (b) Three-dimensional and/or pseudoacinar
have “signet ring” cell features. The differential arrangements of tumor cells
diagnoses include: (c) Lacy cytoplasm with vacuolization
(a) Metastatic adenocarcinoma of the lung (d) Dispersed tumor cells with markedly atypia and
(b) Reactive mesothelial cell hyperplasia enlargement
(c) A metastatic adenocarcinoma from the stomach
(d) All of the above Q-72. A 50-year-old female developed chronic cough and
shortness of breath. The chest x-ray showed bilateral
Q-66. Cytological features of so-called neuroendocrine small opaque infiltrates and left pleural effusion.
tumors may include all of the following, except: The ultrasound-guided thoracentesis was performed.
(a) Fine chromatin pattern (so-called “salt-and-pep- A large amount of fluid is obtained. The initial eval-
per” pattern) uation of the specimen revealed scattered clusters of
(b) Inconspicuous or small nucleoli atypical epithelioid cells, and a metastatic adenocar-
(c) Scant cytoplasm cinoma was suspected. What is the next step of
(d) Nuclear grooves action?
(a) Repeat thoracentesis in few weeks
Q-67. Which type of mucin is commonly seen in (b) Make cell block as usual
mesothelioma? (c) Core biopsy the lung lesions
(a) Neutral (d) Do nothing
(b) Acid (simple or non-sulfated
(c) Acid (simple, mesenchymal) Q-73. An HIV-positive patient developed pleural and peri-
(d) Acid (complex or sulfated, epithelial) cardial effusion. The cytological preparation of the
effusion reveals numerous atypical lymphoid cells.
Q-68. Which type of mucin stain is more specific to dem- A primary effusion lymphoma is suspected. Which
onstrate mucopolysaccharide substances in tissues of the following cytological features can be seen in
(i.e., mucin in adenocarcinoma)? primary effusion lymphoma?
176 Q.K. Li and W.E. Khalbuss

(a) Individual large lymphocyte with enlarged round Q-79. A paracentesis was performed on a patient with
nuclei ascites. On the cell block preparation, it revealed
(b) Coarse chromatin and prominent nucleoli three-dimensional clusters of tall columnar cells with
(c) Abundant basophilic cytoplasm on the Diff-Quik hyperchromatic nuclei, coarse chromatin, prominent
stain nucleoli, irregular nuclear membrane, and feathering
(d) All of the above cytoplasm. Which one is the most likely primary site
of the tumor?
Q-74. In a malignant ascites specimen, tumor cells were (a) Breast
found to have “intranuclear inclusions.” Which one (b) Lung
of the following tumors is the more likely to have an (c) Colon
“intranuclear inclusion” in a fluid cytology? (d) Renal
(a) Renal cell carcinoma
(b) Urothelial cell carcinoma Q-80. Which of the following cytological features is most
(c) Colonic adenocarcinoma useful in the separation of a poorly differentiated
(d) Hepatocellular carcinoma adenocarcinoma from a small cell carcinoma in fluid
cytology specimen?
Q-75. A stage 3 lung cancer patient developed a pleural (a) Prominent nucleoli
effusion. A thoracentesis was performed, and the (b) Fine or coarse chromatin pattern
cytological examination confirmed the diagnosis of (c) Tumor necrosis
metastatic adenocarcinoma of the lung. The oncolo- (d) Organoid and/or rosette arrangement of tumor
gist asked you to send specimen for molecular stud- cells
ies of EGFR and KRAS mutations. What is the next
step of action? Q-81. A patient who had malignant history of desmo-
(a) Tell the clinician to obtain tumor tissue from plastic/spindle cell melanoma developed ascites.
primary tumor. In cytological preparations, scattered “atypical
(b) Biopsy specimen is needed for molecular studies. spindle cells” were identified. Which of the fol-
(c) Prepare cell blocks and make sure that they have lowing options is the best one in the differential
adequate tumor cells. diagnosis of a metastatic desmoplastic/spindle cell
(d) It is not necessary to do molecular study on met- melanoma?
astatic lung adenocarcinoma. (a) Using unstained slides to perform IHC markers
S100, HMB45, and melanin A
Q-76. In cytological preparation from ascites, the finding (b) Making cell block to perform IHC markers
of psammoma bodies is suggestive of which of the S100, HMB45, and melanin A
following diagnoses? (c) Making cell block to perform IHC marker of
(a) Ovarian carcinoma melanin A only
(b) Mesothelial cell hyperplasia (d) Making cell block to perform IHC marker of
(c) Endosalpingiosis S100 only
(d) All of the above
Q-82. A female patient with a breast mass developed a
Q-77. Which of the features is not seen in a metastatic pleural effusion. All of the following features can be
squamous cell carcinoma in fluid specimens? seen in a metastatic breast carcinoma, except:
(a) Large nuclei with smudgy chromatin (a) Prominent nucleoli
(b) Nuclei with variation in size and shape (b) Intracytoplasmic lumina (magenta body)
(c) Pyknotic nuclei (c) Three-dimensional cell clusters
(d) Cytokeratin formation (d) Naked nuclei

Q-78. An HIV-positive patient developed a lymphomatous Q-83. On a cell block preparation, which one of the fol-
effusion. The differential diagnosis should include lowing positive stains indicates a mesothelioma, but
all of the following, except: not a metastatic adenocarcinoma of the lung?
(a) Burkitt’s or Burkitt’s-like lymphoma (BL) (a) Mucicarmine
(b) Diffuse large B-cell lymphoma (DLBCL) (b) Periodic acid–Schiff (PAS)
(c) Peripheral T-cell non-Hodgkin lymphoma (NHL) (c) Hyaluronic acid–alcian blue
(d) Primary effusion lymphoma (PEL) (d) Cytokeratin
2 Serous Fluid Cytopathology 177

Q-84. In a cytological preparation of a pleural effusion from Q-90. The patient with a mediastinal mass has presented
a patient who has breast cancer, the diagnosis of met- with a left pleural effusion. On the cytospin, the spec-
astatic breast carcinoma is made. The characteristic imen reveals “atypical small blue cells.” The main
cytological features include all of the following, differentiation diagnosis of the pleural effusion is
except: small cell lung carcinoma versus lymphoma. Of the
(a) Hyperchromatic nuclei and prominent following tests/markers, which one is the most useful
nucleoli for the differential diagnosis?
(b) Intranuclear inclusion (a) Immunostains of TTF1 and BerEP4
(c) Three-dimensional cell clusters with smooth (b) Immunostains of AE1/AE3 and TTF1
border (c) Immunostains of CD20 and CD5
(d) Bipolar nuclei (d) Immunostains of synaptophysin, chromogranin,
CD56, and CD45
Q-85. A fluid cytological preparation characteristic of a
metastatic small cell carcinoma includes all of the Q-91. A fluid cytological preparation characteristic of
following, except: “benign” nuclear enlargements and/or reactive
(a) Hyperchromatic nuclei and prominent nucleoli nuclear changes in mesothelial cells includes all of
(b) Scant cytoplasm the following, except:
(c) Three-dimensional cell clusters with nuclear (a) Large nuclei in a cell with normal N/C ratio
molding (b) Irregular nuclear membrane and hyperchromasia
(d) Individual tumor cells with fine chromatin (c) Two-dimensional cell clusters with intercellular
“window”
Q-86. In a fluid cytological preparation, the differential (d) Prominent nucleoli
diagnosis of lymphocytic effusion includes which of
the following? Q-92. Which one of the following procedures has the
(a) Tuberculosis highest sensitivity in the diagnosis of malignant
(b) Metastatic carcinoma pleural lesions as well as malignant effusion?
(c) Small lymphocytic lymphoma/chronic lympho- (a) Thoracentesis
cytic leukemia (b) CT-guided transthoracic needle biopsy
(d) All of the above (c) “Blind” transthoracic needle biopsy
(d) Ultrasound-guided transthoracic needle biopsy
Q-87. In a fluid cytological preparation, characteristics of a
metastatic small cell carcinoma were found. Which Q-93. In a patient with pleural effusion, a cytological
of the following IHC markers has better sensitivity specimen is routinely obtained by thoracentesis or
for detection of small cell carcinomas? ultrasound- and/or CT-guided thoracentesis. Which
(a) TTF1 one of the procedures has the highest specificity in
(b) Synaptophysin the diagnosis of malignant pleural effusion?
(c) Chromogranin A (a) Thoracentesis
(d) CD56 (b) CT-guided transthoracic thoracentesis
(c) “Blind” transthoracic needle biopsy
Q-88. TTF1 is always positive in small cell lung carcinomas (d) Ultrasound-guided thoracentesis
(SCLCs). It can be used to distinguish the origin of the
tumor, i.e., pulmonary versus extrapulmonary origin. Q-94. In a malignant fluid specimen, the appearance of
True or false? two- and three-dimensional tumor cell clusters is only
(a) True seen in adenocarcinomas but not in squamous cell
(b) False carcinomas. True or false?
(a) True
Q-89. Which of the following tumors has a weak punctate (b) False
(“dotlike”) labeling for cytokeratins, including
AE1/AE3 and CAM 5.2? Q-95. Pleural effusion was found in a patient with rheuma-
(a) Lung adenocarcinoma toid arthritis. The diagnostic features of rheumatoid
(b) Small cell lung carcinoma pleuritis include:
(c) Metastatic colonic adenocarcinoma to the lung (a) Multinucleated giant cells
(d) Prostate adenocarcinoma (b) Characteristic “granular” debris
178 Q.K. Li and W.E. Khalbuss

(c) Scattered lymphocytes (b) LE cells contain denatured nuclear material


(d) All of the above with a glassy and homogeneous
appearance.
Q-96. Which type of cells is usually not present in a rheuma- (c) LE cells can be identified in the majority of
toid pleuritis? lupus pleuritis patients.
(a) Multinucleated giant cells (d) LE cells contain nuclear material with visible
(b) Lymphocytes chromatin structures, such as coarse chromatin
(c) Mesothelial cells granules.
(d) Clusters of histiocytes
Q-99. In a fluid specimen, which one of the following features
Q-97. Charcot–Leyden crystals can be found in: is indicative of a malignant effusion?
(a) Asthma (a) Numerous large clusters of cells
(b) Eosinophilic effusion (b) Lacunae spaces on cell block preparation
(c) Allergic sinusitis (c) Presence of “second population” cells
(d) All of the above (d) All of the above

Q-98. All of these statements/descriptions of the lupus Q-100. The finding of clear spaces (lacunae) either around
erythematosus (LE) cell are correct in lupus pleuritis, individual cells or around clusters of cells is charac-
except: teristic of malignant effusion. True or false?
(a) The characteristic cell is the lupus erythematosus (a) True
(LE) cells. (b) False
2 Serous Fluid Cytopathology 179

2.3 Answers and Discussion three-dimensional or acinar/papillae clusters, with


of Image-Based Questions 1–50 high N/C ratio, prominent nucleoli, lacy cytoplasm,
and cytoplasmic vacuolization. The size of tumor
A-1. (c) Collagen ball cells is larger than those in breast carcinomas.
The finding of a collagen ball in peritoneal washing Reactive mesothelial cells have “windows” between
specimens is not uncommon. Collagen balls are cells. Metastatic melanomas reveal dispersed large
composed of collagen which is covered with a thin tumor cells with prominent nucleoli, binucleation,
layer of benign mesothelial cells. One study has and cytoplasmic melanin pigments.
shown that collagen balls were found in 1.6 % of
peritoneal washings and 5.8 % of pelvic washings, A-4. (c) Ewing’s sarcoma
respectively. Collagen balls are a nonspecific finding Ewing’s sarcoma is a malignant small, round, blue
and most probably originate on the surface of the cell tumor. It predominantly involves males and usu-
ovary. It can be found in benign conditions and/or ally presents in childhood or early adulthood, with a
from detached fragments of a papillary ovarian neo- peak between 10 and 20 years of age. The tumor can
plasm. Common differential diagnoses of collagen occur anywhere in the body but most commonly in
balls include metastatic mucinous adenocarcinomas, the pelvis and proximal long tubular bones. The cyto-
papillary serous carcinomas of the ovary, and meso- logical preparations reveal dispersed individual small
theliomas. In all these malignant lesions, tumor cells blue cells with two types of tumor cells (tigroid
reveal high N/C ratio, hyperchromatic nuclei, irregu- appearance), i.e., tumor cells with pale nuclei and fine
lar nuclear membrane, and prominent nucleoli. chromatin, and degenerated tumor cells with hyper-
chromatic nuclei and smudged chromatin. Nuclear
A-2. (a) Lupus erythematosus (LE) cells molding is common. The cytoplasm is scant and with
The lupus erythematosus (LE) cell is a neutrophil or vacuolization. Ewing’s sarcoma cells are positive for
macrophage that has phagocytized (engulfed) the CD99 and MIC2 and negative for CD45. The most
denatured nuclear material (hematoxylin body) of common translocation, present in approximately
another cell. Denatured nuclear material in the neu- 90 % of Ewing’s sarcoma cases, is t(11;22)(q24;q12).
trophil/macrophage has a glassy and homogeneous Other translocations are at t(21;22) and t(7;22).
appearance; it may push the nuclei to the peripheral The differential diagnosis includes lymphoma, alveolar
location. In contrast, tart cells contain ingested rhabdomyosarcoma, desmoplastic small round cell
nuclei with a visible chromatin rather than homoge- tumor, small cell carcinoma, and others.
neous appearance. Reactive mesothelial cells have
“windows” between cells, and mucin-producing A-5. (d) Mesothelioma
cells, such as a mucin-producing adenocarcinoma, The cytological finding of mesotheliomas reveals
have cytoplasmic vacuole and lacy cytoplasm. numerous three-dimensional clusters with scalloped
(knobby) edges, characteristic of mesotheliomas.
A-3. (d) Metastatic breast lobular carcinoma Tumor cells have round centrally placed nuclei,
The diagnosis of this pericardial effusion is meta- coarse chromatin, prominent nucleoli, dense cyto-
static breast carcinoma. In fluid preparation, meta- plasm with peripheral “halo,” and intercellular “win-
static breast carcinoma may reveal several patterns. dows.” Reactive mesothelial cells show a spectrum of
Tumor cells of metastatic ductal carcinoma may changes ranging from normal to markedly atypical
form three-dimensional “cannon ball” clusters. In cells. In adenocarcinoma and poorly differentiated
cases of poorly differentiated (high-grade) ductal carcinoma, tumor cells form three-dimensional clus-
carcinomas, specimens consist of predominantly dis- ters with smooth edges. They have hyperchromatic
persed individual cells, whereas in metastatic lobular nuclei, irregular nuclear membrane, prominent nucle-
carcinomas, tumor cells are predominantly presented oli, scant cytoplasm, and without intercellular “win-
as dispersed individual cells. The size of tumor cells dows.” In addition, immunostains of mesothelial cell
is relatively smaller than metastatic adenocarcino- markers (calretinin, GLUT-1, D2-40, WT1, P53) and
mas from the lung and/or GI tract. The presence of adenocarcinoma markers (TTF, Napsin A, BerEP4,
intracytoplasmic lumina, so-called magenta bodies, B72.3, CEA, CD15) may be helpful in the differen-
is also one of the characteristics of a metastatic tial diagnosis. Metastatic melanomas have large dis-
breast carcinoma. In metastatic adenocarcinomas persed tumor cells with large nuclei, prominent
of the lung, columnar tumor cells are arranged in nucleoli, and cytoplasmic melanin pigments.
180 Q.K. Li and W.E. Khalbuss

A-6. (d) Metastatic rhabdomyosarcoma discohesive with clumped chromatin and scant
Rhabdomyosarcoma is commonly seen in children cytoplasm. Lymphoglandular bodies may be identi-
ages one to five and in adolescents. It is also seen fied in the background of the smear. Metastatic
in adults but is rare. The most common anatomic melanoma reveals large discohesive tumor cells
locations for the tumor include the head, neck, and with prominent nucleoli and cytoplasmic pigment.
genitourinary tract. Several different histological
subtypes of rhabdomyosarcoma exist, each of which A-9. (b) Mesothelial cell hyperplasia
has different clinical and morphological characteris- The specimen reveals clusters of reactive mesothelial
tics. Prognosis and clinical behavior of the tumor also cells with psammoma bodies. Reactive mesothelial
partially depend on histological subtype. The most cells may show nuclear atypia; however, the normal
common subtypes are embryonal rhabdomyosarcoma or low N/C ratio is usually maintained. The findings
and alveolar rhabdomyosarcoma. The cytology of reactive atypia represent a spectrum of cellular
sample consists predominantly of dispersed cells. changes that range from normal to markedly atypical.
The tumor cells are intermediate in size and admixed Psammoma bodies can be found in both benign and
with scattered large bizarre cells. Tumor cells reveal malignant conditions. Reactive mesothelial changes
hyperchromatic nuclei, prominent nucleoli, and dense can be caused by a variety of conditions, such as che-
cytoplasm (indicative of muscle differentiation). motherapy and radiation therapy, inflammation,
MyoD1 and myogenin are positive for tumor cells. chronic irritation, and others. In mesothelioma, tumor
cells form numerous three-dimensional clusters.
A-7. (d) Lymphomatous effusion most consistent with An intercellular “window” can be seen in both reac-
malignant lymphoma tive mesothelial cells and mesothelioma. In ovarian
Monomorphous population of lymphocytes is usu- serous carcinomas, tumor cells form three-dimensional
ally seen in lymph proliferative disorders and/or or papillary clusters with hyperchromatic nuclei,
lymphomas, particularly with the presence of apop- irregular nuclear membranes, prominent nucleoli,
tosis. It represents a monoclonal proliferation of and coarse chromatin. In mesotheliomas, tumor
lymphoma cells. Other cytological features of cells form numerous three-dimensional clusters
lymphoma include discohesive tumor cells with with hyperchromatic nuclei, irregular nuclear mem-
hyperchromatic nuclei, clumped (soccer-ball-like) branes, and coarse chromatin.
chromatin, irregular nuclear membrane, nucleoli,
and scant cytoplasm. In small cell carcinomas, A-10. (d) Lymphocytosis (lymphocytic effusion)
tumor cells form clusters or dispersed individual Lymphocytosis (lymphocytic effusion) is the condi-
cells. Nuclear crowding and molding are common. tion in which most of nucleated cells are lympho-
Metastatic poorly differentiated adenocarcinomas cytes. It is a nonspecific finding and can be caused
reveal clusters of large tumor cells with hyperchro- by a variety of diseases, such as tuberculosis, infec-
matic nuclei, coarse chromatin, prominent nucleoli, tions, lymphoma, and other malignancies. In malig-
and cytoplasmic vacuoles. Metastatic melanomas nancy, lymphocytic effusion is caused by the
reveal dispersed individual large tumor cells with obstruction of lymphatic outflow by the tumor or
eccentrically located nuclei, prominent nucleoli, and peritumoral lymphocytic reaction. Repeat tap and
cytoplasmic melanin pigment. cytological evaluation may increase the chance of
finding tumor cells. In lymphoma, particularly in
A-8. (b) Metastatic small cell lung carcinoma small lymphocytic lymphoma/chronic lymphocytic
In small cell carcinomas, tumor cells form two- or leukemia, tumor cells reveal hyperchromatic nuclei
three-dimensional clusters or dispersed individual with clump chromatin. Flow cytometry and immu-
cells and reveal hyperchromatic nuclei, fine granular nostain of B-cell markers may help in the diagnosis.
(salt-and-pepper) chromatin, and scant cytoplasm. In Other causes, such as status postcoronary artery
addition, nuclear crowding, molding, apoptotic body, bypass, may also induce a lymphocytic effusion.
and mitotic figures are common features. The back- In benign lymphocytosis, lymphocytes reveal
ground of the smear may also reveal “blue stripes” euchromatic nuclei with evenly distributed chroma-
(indicative of breakdown of nuclear material). In ade- tin and inconspicuous nucleoli.
nocarcinomas, tumor cells form acini and/or three-
dimensional clusters and reveal hyperchromatic A-11. (c) Metastatic melanoma
nuclei, vesicular or coarse chromatin, prominent In metastatic melanoma, tumor cells are usually
nucleoli, and vacuolated cytoplasm (indicative of arranged in loosely cohesive clusters or dispersed
mucin production). In lymphoma, tumor cells are individual cells. Nuclei of tumor cells are eccentrically
2 Serous Fluid Cytopathology 181

located and highly variable in size with finely to clusters. In squamous cell carcinomas, tumor cells
coarsely granular chromatin and single prominent have enlarged nuclei, smudgy chromatin, and dense
cherry-red nucleoli. The cytoplasm tends to be cytoplasm. In nonkeratinizing squamous cell carci-
abundant and may or may not contain melanin nomas, prominent nucleoli may be present and can
pigment. The N/C ratio may not be high. Melanin be confused with an adenocarcinoma. Squamous
pigment appears coarsely granular and dark brown carcinoma cells have distinct cell borders due to the
on the Papanicolaou stain. Other characteristic fea- production of cytokeratin. In contrast, cytoplasm of
tures include plasmacytoid appearance, binucleation, adenocarcinoma is lacy and vacuolated.
and intranuclear inclusions. The differential diagnosis
of malignant melanoma in effusion includes poorly A-15. (b) Reactive mesothelial cells
differentiated carcinoma and sarcoma. In reactive mesothelial cells, nuclear atypia can be
quite striking; however, the normal or low N/C ratio is
A-12. (d) Mesothelioma usually maintained in reactive atypia. The findings of
The cytological findings of mesothelioma include reactive atypia represent a spectrum of cellular
that tumor cells form numerous three-dimensional changes that range from normal to markedly atypical.
clusters with scalloped (knobby) edges. Tumor cell Reactive mesothelial changes (reactive atypia) can be
has round or oval centrally placed nuclei, coarse caused by a variety of conditions, such as chemother-
chromatin, prominent nucleoli, dense cytoplasm apy and radiation therapy, inflammation, chronic irri-
with peripheral “halo,” and intercellular “windows.” tation, and others. In mesothelioma, tumor cells form
Reactive mesothelial cells show a spectrum of numerous three-dimensional clusters. An intercellu-
changes ranging from normal to markedly atypical lar “window” can be seen in both reactive mesothelial
cells. In adenocarcinomas and poorly differentiated cells and mesothelioma.
carcinomas, tumor cells form three-dimensional
clusters with smooth edges. Tumor cells reveal A-16. (d) Metastatic signet ring cell carcinoma
hyperchromatic nuclei, irregular nuclear membrane, The diagnosis is metastatic signet ring cell carcinoma.
prominent nucleoli, and lacy or vacuolated The differential diagnosis of so-called “signet ring”
cytoplasm, without intercellular “windows.” cells in effusion cytology is broad, including both
Immunostains of mesothelial cell markers (cal- benign and malignant conditions. Benign reactive
retinin, D2-40, WT1, P53) and adenocarcinoma mesothelial cells may have “signet ring” cell features.
markers (TTF, Napsin A, BerEP4, B72.3, CEA, A careful evaluation of the nuclear features is impor-
CD15) may be helpful in the differential diagnosis. tant for distinguishing benign from malignant lesions.
Metastatic melanomas have dispersed tumor cells Both metastatic adenocarcinomas of the lung and
with large nuclei, prominent nucleoli, and cytoplas- stomach can present “signet ring” cell morphology.
mic melanin pigments. The nuclear features of a carcinoma include large
nuclei, hyperchromatic chromatin, irregular nuclear
A-13. (c) Metastatic melanoma member, and large prominent nucleoli. The immu-
The diagnosis is a metastatic melanoma. Prominent nostains of TTF1, Napsin A, and CDX2 may help in
cherry-red nucleoli and intranuclear pseudoinclu- the differential diagnosis of lung adenocarcinoma
sions are common findings in metastatic melano- from stomach carcinoma. In a metastatic lung adeno-
mas. Nuclei of tumor cells are eccentrically located carcinoma, TTF1 and Napsin A are usually positive,
(plasmacytoid appearance) and highly variable in but CDX2 is negative.
size with finely to coarsely granular chromatin.
The cytoplasmic melanin pigment is in dark brown A-17. (a) Metastatic papillary renal cell carcinoma
color on the Papanicolaou stain. Another character- Papillary renal cell carcinoma is the second most
istic feature is the presence of binucleated cells. The common carcinoma of the renal tubules and has been
differential diagnosis of intranuclear pseudoinclu- characterized genetically. There are two histomor-
sions in malignant tumors includes papillary thyroid phologic types. Type 1 tumor consists of papillae and
carcinoma, breast carcinoma, lung adenocarcinoma, tubular structures covered by small cells with pale
meningioma, and others. cytoplasm and characterized by small oval nuclei
with inconspicuous nucleoli, frequent glomeruloid
A-14. (a) Metastatic squamous cell carcinoma papillae, foamy macrophages in papillary cores, and
The diagnosis is metastatic squamous cell carcinoma. psammoma bodies. Type 2 tumor consists of papillae
In fluid cytology, tumor cells of metastatic squamous covered by large cells with abundant eosinophilic
cell carcinomas may be rounded up and form loose cytoplasm and characterized by pseudostratification
182 Q.K. Li and W.E. Khalbuss

and large spherical nuclei with prominent nucleoli In histology sections, tumor cells typically reveal as
and psammoma bodies, but it is uncommon to see small, round and blue cells with Homer–Wright
foamy macrophages in papillary cores. Type 2 tumors rosettes. In fluid cytology, the specimen is cellular
are larger, more common in patients younger than age and consists of discohesive small cells with hyper-
40, and more frequent in stage 3 and 4 tumors. chromatic nuclei and inconspicuous nucleoli.
The reactivity of cytokeratin 7 is specific for papillary Necrotic debris is common. The chromatin is finely
renal cell carcinomas. granular and has “salt-and-pepper” appearance.
Rosettes may be seen on cell block preparation.
A-18. (a) Metastatic adenocarcinoma of the lung A variety of immunohistochemical stains are used
Metastatic adenocarcinomas of the lung may have a to distinguish neuroblastomas from other tumors,
variety of appearances. In lung adenocarcinomas, such as rhabdomyosarcomas, Ewing’s sarcoma,
particularly well-differentiated adenocarcinomas, lymphoma, and Wilms’ tumor. Neuroblastomas are
tumor cells form acinar/papillary arrangements, or immunoreactive for neuron-specific enolase (NSE),
three-dimensional clusters. Tumor cells are interme- synaptophysin, S100, and glial fibrillary acidic pro-
diate or large in size and have hyperchromatic nuclei tein (GFAP). Cytogenetic studies may demonstrate
with coarse granular chromatin, prominent nucleoli, alterations of chromosome 1.
and vacuolated or clear cytoplasm. An intercellular
“window” characteristic of mesothelial cells is not A-21. (c) Metastatic hepatocellular carcinoma
seen in adenocarcinomas. Small cell carcinomas In metastatic hepatocellular carcinoma, particularly
have fine granular (salt-and-pepper) chromatin, a poorly differentiated tumor, numerous pleomor-
nuclear crowding, and molding. phic dispersed individual cells with hyperchromatic
nuclei, prominent nucleoli, and dense granular cyto-
A-19. (d) CD56 plasm are revealed. The identification of cytoplas-
This is a small cell lung carcinoma. CD56 is consid- mic bile pigment or lipofuscin pigment is helpful.
ered to be the most sensitive neuroendocrine marker In poorly differentiated adenocarcinoma, tumor
for small cell lung carcinoma. Approximately 25 % cells have acinar arrangements with vesicular chro-
of small cell lung carcinomas are negative for both matin and lacy cytoplasm. In poorly differentiated
synaptophysin and chromogranin A, but most of squamous cell carcinomas, tumor cells have
these tumors are positive for CD56. The reactivities smudged nuclei and cytoplasmic keratin. In reactive
for synaptophysin and chromogranin A are typically mesothelial cells, cells reveal reactive atypia.
weak in small cell lung carcinomas. TTF1 is
expressed by thyroid and pulmonary cells and is pos- A-22. (c) Metastatic colonic adenocarcinoma
itive in approximately 70–80 % of small cell lung A metastatic colonic adenocarcinoma has characteris-
carcinomas. Still, 10 % of small cell lung carcinomas tic cytological features, such as tall columnar “picket
are negative for all three commonly used neuroendo- fence” appearance, hyperchromatic pencil-shaped
crine markers, synaptophysin, chromogranin A, and nuclei, coarse chromatin, and prominent nucleoli.
CD56. The presence of “dirty” necrosis can be seen in both
cytospin smear and cell block sections. In breast car-
A-20. (b) Neuron-specific enolase (NSE), synaptophysin, cinomas, tumor cells form tight three-dimensional
S100, and glial fibrillary acidic protein (GFAP) “cannon ball” clusters. The size of tumor cells is
The diagnosis is a metastatic neuroblastoma. smaller than colon adenocarcinomas; they may have
Neuroblastoma is the most common solid cancer in intranuclear inclusions. Both reactive mesothelial cells
infancy and early childhood. Nearly half of neuro- and mesotheliomas have intercellular “windows,”
blastoma cases occur in children younger than two and the edge of the cell cluster is knobby (scalloped
years. The tumor derives from primitive sympathetic appearance).
neural precursors and most frequently involves
the adrenal glands, neck, chest, abdomen, and pel- A-23. (d) Metastatic mucinous adenocarcinoma of the
vis. The great majority of cases are sporadic. About lung
1–2 % of cases run in families. N-myc oncogene Cytological features of mucinous adenocarcinomas
amplification is a common finding in neuroblas- include three-dimensional clusters and acinar and
toma. Other genetic abnormalities include activation papillary groups. Tumor cells reveal large round- or
of HRAS, chromosome deletion or allelic loss at 1p, oval-shaped nuclei, coarse chromatin, irregular nuclear
aberrant expression of TRKA and TRAB, and others. membrane, and prominent nucleoli. The cytoplasm
2 Serous Fluid Cytopathology 183

of tumor cells is vacuolated (indicative of mucin derived from B cells) composed of mononuclear
production). Mucinous material can be seen in the Hodgkin and multinucleated Reed–Sternberg cells
background of cytospin smears and cell block prep- residing in an infiltrate containing a variable mixture
aration. In true papillary adenocarcinoma, numerous of nonneoplastic small lymphocytes, eosinophils,
papillary structures could be identified. A meta- neutrophils, histiocytes, and plasma cells. Large
static colonic adenocarcinoma has hyperchromatic tumor cells (Reed–Sternberg cells) are positive for
pencil-shaped nuclei and tumor necrosis. Reactive CD30 and CD15. In SLL/CLL, it consists of mono-
mesothelial cells reveal an intercellular “window” morphous small lymphocytes.
and reactive atypia.
A-27. (c) Reactive mesothelial cells
A-24. (d) Metastatic squamous cell carcinoma The specimen consists of epithelioid cells and
In metastatic squamous cell carcinomas, the cyto- inflammatory cells. The epithelioid cells are reactive
logical features of tumor cells include large nuclei mesothelial cells, and some of them show signet
with smudgy chromatin, nuclei with variation in size ring cell morphology. The finding of signet ring cell
and shape, and dense cytoplasm (indicative of cyto- morphology is not uncommon in reactive mesothe-
keratin formation). In well-differentiated tumors, lial cells. A careful evaluation of the nuclear features
cytokeratin stains orangeophilic on the Papanicolaou is important for distinguishing benign from malig-
method. Tumor cells are usually rounded up in the nant conditions. The nuclear features of a signet ring
fluid specimens. The formation of two- and three- cell carcinoma include large nuclei, hyperchromatic
dimensional tumor cell clusters is not uncommon, chromatin, irregular nuclear member, and large
particularly in poorly differentiated squamous cell prominent nucleoli. All these features are absent in
carcinomas. In addition, prominent nucleoli can be benign reactive mesothelial cells with signet ring
seen in poorly differentiated squamous cell carcino- morphology.
mas and should not be confused with poorly differ-
entiated adenocarcinomas. A-28. (b) Metastatic squamous cell carcinoma
In metastatic squamous cell carcinomas, the cyto-
A-25. (b) Primary effusion lymphoma logical features of tumor cells include large nuclei
Primary effusion lymphoma (PEL) is a rare HIV- with smudgy chromatin, nuclei with variation in size
associated non-Hodgkin lymphoma (NHL) that and shape, and dense cytoplasm (indicative of cyto-
accounts for approximately 4 % of all HIV-associated keratin formation). In well-differentiated tumors,
NHL. In PEL, tumor cells are large and have hyper- cytokeratin stains orangeophilic on the Papanicolaou
chromatic round nuclei, prominent nucleoli, and method. Tumor necrosis may be found in the smear
varying amounts of cytoplasm. The cells show a and cell block preparation, too.
range of appearances, from immunoblastic (round
nuclei with central prominent nucleoli) to plasma- A-29. (d) Metastatic serous carcinoma
blastic (eccentric nuclei with abundant cytoplasm, The diagnosis is metastatic serous carcinoma. The
sometimes containing a perinuclear clearing) to ana- specimen reveals large papillary clusters of malignant
plastic (very large round or polygonal cells with cells and psammoma bodies. Psammoma bodies can
bizarre, pleomorphic nuclei). Anaplastic forms may be found in both benign and malignant conditions.
also contain multinucleated and/or Reed–Sternberg- In serous carcinomas, tumor cells form three-dimensional
like cells. Immunophenotypically, PEL cells reveal a or papillary clusters with hyperchromatic nuclei,
“null” lymphocyte phenotype (i.e., CD45 is irregular nuclear membrane, prominent nucleoli, and
expressed), but routine B-cell (including surface and coarse chromatin. Reactive mesothelial cells reveal a
cytoplasmic immunoglobulin, CD19, CD20, CD79a) spectrum of reactive changes and intercellular “win-
and T-cell (CD3, CD4, CD8) markers are negative. dows” and the presence of psammoma bodies. In
colonic adenocarcinomas, tumor cell forms numer-
A-26. (d) Hodgkin lymphoma ous three-dimensional clusters with hyperchromatic
The cytology reveals scattered large atypical cells elongated nuclei and tumor necrosis. In squamous
with binucleation and prominent nucleoli (Reed– cell carcinomas, tumor cells are arranged in loose
Sternberg cells) in a background of mixed inflam- clusters or dispersed individual cells with variable
matory cells, and findings are characteristic of shaped nuclei and dense cytoplasm with cytokeratin
Hodgkin lymphoma. Classic Hodgkin lymphoma is formation. Psammoma bodies are not seen in these
a monoclonal lymphoid neoplasm (in most instances two types of carcinomas.
184 Q.K. Li and W.E. Khalbuss

A-30. (c) Reactive mesothelial cells form numerous large clusters with hyperchromatic
Reactive cellular changes can happen in a variety nuclei, clumped chromatin, and irregular nuclear
of settings, such as radiation therapy and chemo- membrane. In colonic adenocarcinoma, tumor cells
therapy. Reactive mesothelial cells may form cohe- form three-dimensional clusters with elongated
sive sheets and/or two-dimensional clusters of cells. nuclei and tumor necrosis. In serous carcinomas,
These cells can be larger in size and have enlarged tumor cells have papillary arrangements, hyperchro-
nuclei, but the normal N/C ratio is maintained. matic nuclei, irregular nuclear membrane, promi-
The differential diagnosis of reactive mesothelial nent nucleoli, scant cytoplasm, and without
cells includes mesotheliomas and carcinomas, par- intercellular “windows.”
ticularly mesotheliomas. In mesotheliomas, tumor
cells form three-dimensional clusters with knobby A-34. (b) Metastatic dysgerminoma
edges. They have hyperchromatic nuclei, irregular The diagnosis is a metastatic dysgerminoma.
nuclear membranes, and prominent nucleoli. Dysgerminoma of the ovary and its counterpart semi-
noma of the testis present as discohesive clusters and
A-31. (b) Calretinin, TTF, and BerEP4 single cells that resemble enlarged mesothelial cells,
The cytological preparation shows a metastatic ade- although their cytoplasm is generally scant and clear.
nocarcinoma. In pleural effusion, the most common The nuclei are large and round with coarse chromatin
metastatic adenocarcinoma is from the lung. In lung and prominent nucleoli. The tumor cells have abun-
adenocarcinomas, tumor cells form three-dimensional dant clear cytoplasm. The background may contain
clusters with smooth edges and without intercellular numerous lymphocytes. But, the “tigroid’ appearance
“windows.” They have hyperchromatic nuclei, irreg- that is so characteristic of seminomas/dysgerminoma
ular nuclear membrane, prominent nucleoli, and on fine-needle aspiration specimens (owing to the
scant cytoplasm. The immunostains of calretinin abundance of glycogen from disruption of the deli-
(negative), TTF1 (positive), and BerEP4 (positive) cate cytoplasm) is not apparent in fluid specimens.
would confirm the diagnosis of a metastatic adeno- The differential diagnosis includes poorly differenti-
carcinoma of the lung. ated carcinoma, melanoma, anaplastic large cell lym-
phoma, and epithelioid mesothelioma. The tumor
A-32. (a) Metastatic mucinous adenocarcinoma cells of dysgerminoma are positive for periodic acid–
The diagnosis is metastatic mucinous adenocarci- Schiff (PAS) stain, placental alkaline phosphatase
noma. As seen in adenocarcinomas, tumor cells (PLAP), CD117, and OCT4, but usually negative for
form three-dimensional clusters. They have hyper- cytokeratin (CAM5.2 and/or AE1/AE3) and epithe-
chromatic nuclei, irregular nuclear membrane, and lial membrane protein (EMA).
prominent nucleoli. The striking cytological feature
of mucinous adenocarcinomas is the cytoplasm vac- A-35. (c) Reactive mesothelial cells
uoles and clear cytoplasm (indicative of cytoplasmic Reactive mesothelial cells can form cohesive sheets
mucin production). Reactive mesothelial cells form and/or two-dimensional clusters of cells. These cells
sheets or loose clusters; they have dense cytoplasm can be larger in size and have enlarged nuclei, but
and intercellular “windows.” Metastatic melanomas the normal N/C ratio is maintained. Multinucleation
reveal dispersed large tumor cells with prominent and prominent nucleoli are common. Intercellular
nucleoli and cytoplasmic melanin pigment. In meta- “windows” are the feature of mesothelial cells. The
static small cell carcinomas, tumor cells have fine differential diagnosis of reactive mesothelial cells
granular (salt-and-pepper) chromatin, nuclear includes mesothelioma and carcinoma, particularly
crowding, and molding. a well-differentiated adenocarcinoma. In adenocar-
cinomas, tumor cells form three-dimensional clus-
A-33. (c) Reactive mesothelial cells ters with smooth edges. They have hyperchromatic
The specimen consists of small clusters of cuboid nuclei, irregular nuclear membrane, prominent
cells with lumen formation and dispersed individual nucleoli, scant cytoplasm, and without intercellular
cells, characteristic of reactive mesothelial cells. “windows.” In mesothelioma, tumor cells form
Reactive mesothelial cell may be larger in size and numerous large three-dimensional clusters.
have enlarged nuclei, but the normal N/C ratio is
maintained. Multinucleation and prominent nucleoli A-36. (c) Reactive mesothelial cells
may be also seen. In mesothelioma, tumor cells See answer of Q-35.
2 Serous Fluid Cytopathology 185

A-37. (b) Reactive mesothelial cells A-41. (b) Reactive mesothelial cells
See answer of Q-35. Reactive mesothelial cells show a spectrum of
changes, including sheets and/or cohesive clusters
A-38. (c) Endometriosis of cells, and they are larger in size with enlarged
The diagnosis of this pelvic washing specimen is nuclei, but the normal N/C ratio is maintained.
endometriosis. Endometriosis is typically seen dur- Multinucleation and prominent nucleoli are com-
ing the reproductive years; it has been estimated that mon. In mesotheliomas, tumor cells form numerous
endometriosis occurs roughly in 10 % of women. large three-dimensional clusters. Tumor cells have
The most common location is the ovary. On a fluid hyperchromatic nuclei, irregular nuclear membrane,
specimen, benign columnar cells (endometrial prominent nucleoli, and scant cytoplasm. In adeno-
glands), hemosiderin-laden macrophages, spindle carcinomas and serous carcinomas, tumor cells form
cells (endometrial stroma), and debris are common three-dimensional or papillary clusters with smooth
findings. The specimen can be quite bloody. Older edges. They have hyperchromatic nuclei, irregular
lesions may display no glands but hemosiderin nuclear membrane, prominent nucleoli, vacuolated
deposits. cytoplasm, and without intercellular “windows.”

A-39. (b) Reactive mesothelial cells A-42. (c) Metastatic melanoma


Reactive mesothelial cells may form sheets and/or The diagnosis is a metastatic melanoma. In metastatic
cohesive clusters of cells. These cells can be larger in melanomas, tumor cells are usually arranged in
size and have enlarged nuclei, but the normal N/C loosely cohesive clusters or dispersed individual
ratio is maintained. Multinucleation and prominent cells. Nuclei of tumor cells are eccentrically located
nucleoli are common. In mesotheliomas, tumor cells and highly variable in size with finely to coarsely
form numerous large three-dimensional clusters. granular chromatin and single prominent cherry-red
Tumor cells have hyperchromatic nuclei, irregular nucleoli. The cytoplasm tends to be abundant and
nuclear membrane, prominent nucleoli, scant cyto- may or may not contain melanin pigment. The N/C
plasm, and intercellular “windows.” In adenocarci- ratio may not be high. Melanin pigment appears
nomas, tumor cells form three-dimensional clusters coarsely granular and dark brown on the Papanicolaou
with smooth edges. They have hyperchromatic stain. Other characteristic features include plasmacy-
nuclei, irregular nuclear membrane, prominent toid appearance, multinucleation, binucleation, and
nucleoli, vacuolated cytoplasm, and without intercel- intranuclear inclusions. The differential diagnosis
lular “windows.” of malignant melanomas in effusion includes poorly
differentiated carcinoma, lymphoma, and sarcoma.
A-40. (a) Metastatic adenocarcinoma of the lung
The diagnosis is a metastatic adenocarcinoma of the A-43. (a) Rheumatoid pleuritis
lung. The specimen reveals many variably sized The diagnosis is rheumatoid pleuritis. The rheuma-
three-dimensional clusters and scattered psammoma toid pleuritis occurs in approximately 2–3 % of
bodies, characteristic of lung adenocarcinoma. In patients with rheumatoid arthritis. The pleural
adenocarcinomas, tumor cells form three-dimensional effusion can be unilateral or bilateral. Cytological
clusters with smooth edges. They have hyperchro- features include abundant granular debris, multi-
matic nuclei, irregular nuclear membrane, promi- nucleated giant cells, and scattered lymphocytes.
nent nucleoli, vacuolated cytoplasm, and without The presence of granular debris is characteristic of
intercellular “windows.” In benign conditions such rheumatoid pleuritis. Granular debris is amorphous
as mesothelial cell hyperplasia and endosalpingio- material and forms variable sized green- or pink-
sis, the presence of psammoma bodies is a common red-colored clumps by the Papanicolaou stain. It is
finding. In ovarian serous carcinomas, tumor cells believed that the amorphous material is different
form three-dimensional or papillary clusters with from fibrin. Diagnosis of rheumatoid pleuritis relies
hyperchromatic nuclei, irregular nuclear on the characteristic cytological findings in an exu-
membranes, prominent nucleoli, and coarse dative pleural fluid.
chromatin. In mesotheliomas, tumor cells form
numerous three-dimensional clusters with hyper- A-44. (c) Reactive mesothelial cells
chromatic nuclei, irregular nuclear membranes, and The diagnosis is reactive mesothelial cells. The
coarse chromatin. specimen consists of small clusters of cuboid cells
186 Q.K. Li and W.E. Khalbuss

with lumen formation and dispersed individual cells A-47. (b) Reactive mesothelial cells
characteristic of reactive mesothelial cells. Reactive The diagnosis is reactive mesothelial cells. Reactive
mesothelial cells may be larger in size and have mesothelial cells show a spectrum of changes,
enlarged nuclei, but the normal N/C ratio is main- including sheets and/or cohesive clusters of cells;
tained. In mesotheliomas, tumor cells form numer- they are larger in size with enlarged nuclei, but the
ous three-dimensional clusters with hyperchromatic normal N/C ratio is maintained. Multinucleation and
nuclei, clumped chromatin, and irregular nuclear prominent nucleoli are common. In mesotheliomas,
membrane. In adenocarcinomas, tumor cells form tumor cells form numerous large three-dimensional
three-dimensional clusters with hyperchromatic clusters. Tumor cells have hyperchromatic nuclei,
nuclei, irregular nuclear membrane, prominent irregular nuclear membrane, prominent nucleoli,
nucleoli, and scant cytoplasm, and without intercel- and scant cytoplasm. In adenocarcinomas and serous
lular “windows.” carcinomas, tumor cells form three-dimensional or
papillary clusters with smooth edges. They have
A-45. (d) Metastatic squamous cell carcinoma hyperchromatic nuclei, irregular nuclear membrane,
The diagnosis is a metastatic squamous cell carci- prominent nucleoli, vacuolated cytoplasm, and
noma. When a squamous cell carcinoma involves without intercellular “windows.”
serosal surfaces and/or tumor cells shed into the
fluid, tumor cells may be rounded up in the fluid and A-48. (a) Metastatic serous carcinoma
with less “squamous” features. Tumor cells of squa- The diagnosis is metastatic serous carcinoma. The
mous cell carcinomas form loose clusters or dis- specimen reveals clusters of atypical epithelial cells
persed individual cells, whereas tumor cells of and psammoma bodies. Psammoma bodies are
adenocarcinoma form three-dimensional clusters or round collections of calcium which is covered with
pseudoacinar arrangements. Tumor cells of squa- a thin layer of benign mesothelial cells. It stains
mous cell carcinomas have hyperchromatic nuclei, purple in color with the Papanicolaou method.
smudgy chromatin, and dense cytoplasm (indicative Psammoma bodies can be found in both benign and
of cytokeratin formation). Tumor cells of adenocar- malignant conditions. In benign conditions such as
cinomas have vesicular nuclei, coarse chromatin, mesothelial cell hyperplasia and endosalpingiosis,
and vacuolated cytoplasm (indicative of mucin pro- the presence of psammoma bodies is a common
duction). The presence of prominent nucleoli can be finding. In malignant conditions such as ovarian
seen in both adenocarcinomas and poorly differenti- serous carcinoma and mesothelioma, numerous
ated squamous cells carcinomas. psammoma bodies can be identified in specimens.
In ovarian serous carcinoma, tumor cells form three-
A-46. (d) Pseudomyxoma peritonei dimensional or papillary clusters with hyperchro-
The diagnosis is pseudomyxoma peritonei. matic nuclei, irregular nuclear membrane, prominent
Pseudomyxoma peritonei is a condition caused by nucleoli, and coarse chromatin. In mesothelioma,
the production of abundant mucin or gelatinous tumor cell forms numerous three-dimensional clus-
material by tumor cells, which fills the abdominal ters with hyperchromatic nuclei, irregular nuclear
cavity. The mucin or gelatinous material stains blue- membrane, and coarse chromatin. Tumor cells of
green or purple color with the Papanicolaou method. squamous cell carcinomas have hyperchromatic
This disease is most commonly caused by an appen- nuclei, smudgy chromatin, and dense cytoplasm
diceal primary cancer, mucinous tumors of the ovary, (indicative of cytokeratin formation); however,
and other gastrointestinal cancers. The primary psammoma bodies are not seen in squamous cell
tumor appears to arise from the MUC2-producing carcinomas.
goblet cells in the appendix. The KRAS (P53) gene
mutations have also be identified in the tumor cells. A-49. (c) Burkitt lymphoma
Cytological findings of pseudomyxoma peritonei Burkitt lymphoma accounts for 30–50 % of child-
include viscous, mucoid fluid that is difficult to hood lymphoma. It is a highly aggressive B-cell
smear (thick), free-floating columnar epithelial cells lymphoma and has three main clinical variants: the
or goblet-like cells, occasional papillae, or fibrovas- endemic, the sporadic, and the immunodeficiency-
cular cores. Since the majority of these tumors in associated variants. The endemic variant occurs in
pseudomyxoma peritonei are low-grade malignancy/ African and Asian children, commonly involves the
borderline tumors, it is recommended to be called jaw or other facial bone and distal ileum, and is
mucinous tumors unless there is surgical pathology associated with Epstein–Barr virus (EBV) infection.
confirmation of a mucinous carcinoma. The sporadic variant (also known as “non-African”)
2 Serous Fluid Cytopathology 187

occurs in the United States and is not associated A-50. (c) Eosinophilic pleural effusion
with the EBV and involves the ileocecal region. The Eosinophilic pleural effusion (EPF) is defined as a
immunodeficiency-associated Burkitt lymphoma nucleated cell count containing more than 10 %
occurs in adults and is usually associated with HIV eosinophils in the effusion specimen. It is estimated
infection or immunocompromised patients. All three that approximately 10 % of exudative pleural effu-
variants have similar morphology. The tumor con- sions are eosinophilic. The mechanism of the devel-
sists of intermediate-sized cells with round nuclei, opment of EPF is still not fully understood. One
coarse chromatin, multiple nucleoli, and scant cyto- hypothesis is that stimulation of pleural mesothelial
plasm. The cytoplasmic vacuolization is a striking cells by injury (such as trauma, air, and blood) leads
feature of the tumor. Numerous tingible body mac- to the production of cytokines, chemokines, and
rophages, apoptosis, and mitoses can be seen. The adhesion molecules that induce the accumulation of
“starry sky” appearance seen under low power is eosinophils. The most common causes of EPF are
due to scattered tingible body macrophages. The pleural irritation or trauma (i.e., hemothorax, pneu-
tumor cells are positive for B-cell markers (CD20, mothorax, thoracic surgery) and malignancy. Other
CD22, CD19) as well as CD10 and BCL6, but gen- causes include infections (i.e., bacteria, fungi,
erally negative for BCL2 and TdT. The c-myc gene mycobacteria, parasites, and viruses) and certain
is commonly involved (80 % of t(8;14), 15 % of medications. EPF also occurs idiopathic in 8–35 %
t(2;8), and 5 % of t(8;22)). of patients.
188 Q.K. Li and W.E. Khalbuss

2.4 Answers and Discussion of Text-Based small clusters or sheets of cells. The cellular arrange-
Questions 51–100 ment may be confused with an adenocarcinoma.
Variation in nuclear size, binucleation, and multinu-
A-51. (d) Numerous large three-dimensional cellular cleation can be found in both reactive mesothelial
clusters cells and adenocarcinoma. The most characteristic
Mesothelial cells may undergo reactive changes in a finding of mesothelial cells is the presence of “win-
variety of clinical settings such as with acute and dow” between cells.
chronic inflammations. The so-called “reactive”
mesothelial cells can have markedly nuclear atypia, A-54. (d) Mesothelioma
including nuclear enlargement and variation, multi- Pleural mesothelioma is the most common form of
nucleation, and prominent nucleoli. These findings the tumor, accounting for roughly 70 % of mesothe-
represent a spectrum of cellular changes that range lioma cases. An individual may be at risk to develop
from normal to atypical in reactive mesothelial cells. mesothelioma if he or she was exposed to asbestos.
Variation in nuclear size, binucleation, and multinu- Pleural effusion is the common clinical presentation
cleation can be also found in malignant mesothelial of mesothelioma. The cytological findings of meso-
cells. Reactive cells may form two- and three- thelioma include that tumor cells form numerous
dimensional cellular clusters or sheets of cells; how- three-dimensional clusters with scalloped (knobby)
ever, the cellular cluster usually contains less number edges. High-power view reveals that tumor cell has
of cells than that of mesothelioma. In contrast, in round centrally placed nuclei, coarse chromatin,
mesotheliomas, tumor cells form “large” three- prominent nucleoli, dense cytoplasm with peripheral
dimensional clusters with more than 20–50 cells. “halo,” and intercellular “windows.” Reactive meso-
thelial cells show a spectrum of changes ranging
A-52. (b) BerEP4, Napsin A, TTF1, Calretinin, WT1, from normal to markedly atypical cells. In adeno-
and P53 carcinoma and poorly differentiated carcinoma,
Mesothelioma is positive for calretinin, P53, and tumor cells form three-dimensional clusters with
WT1, whereas, adenocarcinoma of the lung is posi- smooth edges. They have hyperchromatic nuclei,
tive for TTF, BerEP4, CEA, B72.3, and CD15. irregular nuclear membrane, prominent nucleoli, scant
Among the so-called adenocarcinoma markers, cytoplasm, and without intercellular “windows.”
BerEP4 has the membrane staining pattern; Napsin In addition, immunostains of mesothelial cell
A is a relative specific marker for lung adenocarci- marker (calretinin, D2-40, WT1, P53) and adeno-
noma with a cytoplasmic staining pattern; and TTF1 carcinoma markers (TTF, Napsin A, BerEP4, B72.3,
has nuclear staining pattern whereas, among meso- CEA, CD 15) should be performed to aid in the
thelioma cell markers such as calretinin, WT1, and differential diagnosis.
P53, calretinin has the nuclear staining pattern and
relatively high sensitivity. Cytokeratin markers, A-55. (d) Shared cell borders
including AE1/AE3 and CK7, are positive for both In squamous cell carcinomas, tumor cells have
epithelial and mesothelial cells. CK20 and Ki67 enlarged nuclei, smudgy chromatin, and dense
play no roles in the differential diagnosis of meso- cytoplasm. In poorly differentiated squamous cell
thelioma and adenocarcinoma. The combination of carcinoma, prominent nucleoli may be present and
BerEP4, Napsin A, TTF1, calretinin, WT1, and P53 can be confused with an adenocarcinoma. Squamous
will provide a critical differential diagnosis for carcinoma cells have distinct cell borders due to the
adenocarcinoma and mesothelioma. production of cytokeratin; therefore, shared cell
borders are not a feature of squamous cell carcinomas.
A-53. (b) “Window” between atypical cells It is more often seen in an adenocarcinoma.
Reactive mesothelial changes (reactive atypia) can
be caused by a variety of conditions, such as chemo- A-56. (d) Hyperchromatic nucleoli and clumped
therapy and radiation therapy, inflammation, chronic chromatin
irritation, and others. In reactive mesothelial cells, Reactive mesothelial cell changes are commonly
nuclear atypia can be quite striking, and the normal seen in patients with radiation therapy and chemo-
or low N/C ratio is usually maintained in reactive therapy. The main differential diagnosis of reactive
atypia. These findings represent a spectrum of cellu- mesothelial cells includes mesothelioma and carci-
lar changes that range from normal to atypical. The noma, particularly a well-differentiated adenocarci-
reactive cells may form two- or three-dimensional noma. Reactive mesothelial cell may form cohesive
2 Serous Fluid Cytopathology 189

sheets and two- and three-dimensional clusters. features seen in reactive mesothelial cells. In a
These cells can be larger in size and have enlarged poorly differentiated squamous cell carcinoma,
nuclei, but the normal N/C ratios are maintained. tumor cells have smudgy chromatin and dense
Multinucleation and prominent nucleoli are com- cytoplasm with or without cytokeratin formation.
mon. However, hyperchromatic nuclei, clumped In basaloid carcinoma and small cell carcinoma,
chromatin, and irregular nuclear membrane are not tumor cells are relatively small in size with hyper-
features seen in reactive atypia. The presence of chromatic nuclei, nuclear molding and crowding,
these features indicates malignant lesions. In adeno- and scant cytoplasm.
carcinomas, tumor cells form three-dimensional
clusters with smooth edges. They have hyperchro- A-59. (b) Renal transplantation
matic nuclei, irregular nuclear membranes, promi- Pleural fluid eosinophilia, also called eosinophilic
nent nucleoli, lacy and vacuolated cytoplasm, and pleural effusion (EPF), is defined as a nucleated cell
without intercellular “windows.” count containing more than 10 % eosinophils in the
effusion specimen. It is estimated that approxi-
A-57. (c) Monomorphous population of lymphocytes mately 10 % of exudative pleural effusions are
Monomorphous population of lymphocytes is usually eosinophilic. The mechanism of the development of
seen in lymph proliferative disorders and/or lym- EPF is still not fully understood. One hypothesis is
phomas, particularly in small lymphocytic lympho- that stimulation of pleural mesothelial cells by
mas/chronic lymphocytic leukemia (SLL/CLL). It injury (such as trauma, air, and blood) leads to the
represents a monoclonal proliferation of lymphoma production of cytokines, chemokines, and adhesion
cells. Other cytological features of SLL/CLL include molecules that induce the accumulation of eosino-
discohesive tumor cells with hyperchromatic nuclei, phils. The most common causes of EPF are pleural
clumped (soccer-ball-like) chromatin, irregular irritation or trauma (i.e., hemothorax, pneumotho-
nuclear membrane, inconspicuous nucleoli, and rax, thoracic surgery) and malignancy. Other causes
scant cytoplasm. Polymorphous population of lym- include infections (i.e., bacteria, fungi, mycobacteria,
phocytes is the feature of reactive lymphoid popula- parasites, and viruses) and certain medications.
tion. The presence of lacunar space (i.e., malignant EPF also occurs idiopathic in 8–35 %of patients.
cells are situated in an empty space) on the cell block Although it was common decades ago, the develop-
section is a feature of malignancy such as metastatic ment of EPF in the setting of tuberculosis is now
adenocarcinoma and melanoma. Lymphoglandular considered rare. Renal transplantation has no direct
bodies can be seen in both reactive lymphocytes and connection to EPF.
lymphoma cases.
A-60. (a) True
A-58. (a) Adenocarcinoma with lepidic growth pattern The sensitivity of fluid cytology for detecting malig-
(bronchoalveolar carcinoma) nancy is approximately 60–70 %, whereas the sensi-
Adenocarcinoma with lepidic growth pattern tivity of “blind” biopsy is less than 50 %. The reason
(formerly bronchoalveolar carcinoma) is considered fluid cytology has a higher sensitivity is that it may
a well-differentiated adenocarcinoma. It can be dif- provide more diagnostic cells. The sensitivity of fluid
ficult to distinguish from reactive mesothelial cells, cytology and the detection rate of malignancy may
particularly in a scant specimen. In a well-differentiated be further increased when multiple specimens are
adenocarcinoma, tumor cells are arranged in two- or examined from a clinically suspicious patient.
three-dimensional clusters. They may also form a
honeycomb-like arrangement. Tumor cells are inter- A-61. (c) Air in the pleural space (pneumothorax)
mediate in size and have round hyperchromatic Common causes of EPF are pleural irritation or
nuclei, vesicular chromatin, small nucleoli, and trauma (i.e., hemothorax, pneumothorax, thoracic
foamy cytoplasm. However, no “window” is identi- surgery), malignancy, infections (i.e., bacteria,
fied between tumor cell clusters. Reactive mesothelial fungi, mycobacteria, parasites, and viruses), and
cells may form cohesive sheets and/or two-dimen- certain medications. The reported frequency of pleu-
sional clusters. These cells can be larger in size and ral malignancy in patients with EPF ranges from 6 to
have enlarged nuclei, but the normal N/C ratios are 40 %. Among the malignancies associated with EPF,
maintained. Multinucleation and prominent nucleoli lung cancer is the most common one. EPF also
are common. Hyperchromatic nuclei, clumped occurs idiopathic in 8–35 % of patients. In many case
chromatin, and irregular nuclear membrane are not studies, pneumothorax, particularly the spontaneous
190 Q.K. Li and W.E. Khalbuss

pneumothorax, is the most common cause of EPF A-65. (d) All of the above
and accounts for more than 50 % of cases. The differential diagnosis of so-called “signet ring”
cells in effusion cytology is broad, including both
A-62. (b) Air in the pleural space (pneumothorax) benign and malignant conditions. In benign reactive
Pneumothorax, particularly the spontaneous pneu- mesothelial cells, they can have “signet ring” cell
mothorax, is the most common cause of EPF. An features. Both metastatic adenocarcinomas of the
intense eosinophilic pleuritis occurs within hours lung and stomach can present “signet ring” cell
after air entry into the pleural space. In contrast to morphology. A careful evaluation of the nuclear
the rapid development of EPF after spontaneous features is important for distinguishing benign from
pneumothorax, hemothorax-induced EPF appears malignant lesions. The nuclear features of a carci-
typically by the tenth day, with the development of a noma include large nuclei, hyperchromatic chroma-
peripheral blood eosinophilia days later. Pleural tin, irregular nuclear member, and large prominent
space hemorrhage may contribute to the develop- nucleoli. The immunostains of TTF1, Napsin A,
ment of PFE in the setting of thoracic surgery, thora- and CDX2 may help in the differential diagnosis of
cotomy, chest trauma, benign asbestos pleural lung adenocarcinoma from stomach carcinoma. In a
effusion, and pulmonary infarction. The thoracic metastatic lung adenocarcinoma, TTF1 and Napsin
surgery and biopsy may also result in EPF which A are usually positive, but CDX2 is negative.
typically occurs within 30 days after the procedure, However, in cases of metastatic mucinous adeno-
particularly when the pleura has been excised, such carcinoma of the lung, stains of TTF and CDX2
as internal mammary grafting. Repeated thoracente- may not help, since the TTF may be negative and
sis has been suggested as a cause of EPF; it develops CDX2 may be focally positive in the mucinous
within days but not in hours after the procedure. adenocarcinoma.

A-63. (d) Cirrhosis A-66. (d) Nuclear grooves


The most common cause of eosinophilic ascites is Neuroendocrine tumors (NET) are a spectrum of
cirrhosis. Other causes of eosinophilic effusion neoplasms, arising from various neuroendocrine
include peritoneal irritation or trauma, malignancy, cells. NETs are most often located in the intestine
infections (i.e., bacteria, fungi, mycobacteria, para- and the lungs. Although NETs have different
sites, and viruses), and idiopathic. A rare cause of embryological origin, they have common pheno-
development of eosinophilic ascites is eosinophilic typic characteristics. For example, tumors show
gastroenteritis, which has been increasingly reported immunoreactivity for neuroendocrine markers,
recently. It should be included in the differential including chromogranin, synaptophysin, CD56,
diagnosis of eosinophilic ascites. and neuron-specific enolase (NES) and may secrete
various peptides and hormones. Cytological fea-
A-64. (d) Prominent nucleoli tures of NET are characterized by fine granular
When squamous cell carcinoma involves serosal chromatin (salt-and-pepper chromatin) pattern,
surfaces and/or tumor cells shed into the fluid, the inconspicuous nucleoli, and scant cytoplasm.
distinction of squamous cell carcinoma and adeno- Mitosis may be rare or abundant depending on the
carcinoma can be difficult in some cases, since tumor differentiation of the tumor. Nuclear crowding and
cells may be rounded up in the fluid and with less molding are often seen in small cell carcinoma;
“squamous” features. Tumor cells of squamous cell however, nuclear groove is not a feature of neuro-
carcinoma form loosely clusters or dispersed individ- endocrine tumors. The World Health Organization
ual cells, whereas tumor cells of adenocarcinoma (WHO) classifies neuroendocrine tumors into three
form three-dimensional clusters or pseudoacinar main categories: (1) well-differentiated neuroendocrine
arrangements. In general, tumor cells of squamous tumors with benign behavior, (2) well-differentiated
cell carcinoma have hyperchromatic nuclei, smudgy (low-grade) neuroendocrine carcinomas with
chromatin, and dense cytoplasm (indicative of cyto- low-grade malignant behavior, and (3) poorly
keratin formation). Tumor cells of adenocarcinoma differentiated (high-grade) neuroendocrine carcino-
have vesicular nuclei, coarse chromatin, and vacuo- mas with aggressive clinical course, such as the
lated cytoplasm (indicative of mucin production). large cell neuroendocrine and small cell carcinomas.
The presence of prominent nucleoli can be seen in In addition, cellular proliferative rate (Ki67 labeling)
both adenocarcinoma and poorly differentiated has been considered as a useful marker for the clas-
squamous cells carcinoma. sification of the tumor.
2 Serous Fluid Cytopathology 191

A-67. (c) Acid (simple, mesenchymal) down the differential diagnosis. The spindle cell
Mucin can be produced by both epithelial cells and appearance is not the feature of a poorly differenti-
stroma. Neutral mucin can be found in glands of the ated adenocarcinoma. The cytological features of a
GI tract and in the prostate, and they stain with PAS poorly differentiated adenocarcinoma include three-
but not with alcian blue, colloidal iron, mucicar- dimensional clusters, acinar and papillary groups, or
mine, or metachromatic dyes. There are several honeycomb-like sheets of epithelial cells. Tumor
types of acid mucins. Acid mucin (simple or non- cells have large round- or oval-shaped hyperchro-
sulfated) is the typical mucin of epithelial cells con- matic nuclei, coarse chromatin, irregular nuclear
taining sialic acid, and they stain with PAS, alcian membrane, and prominent nucleoli. The cytoplasm
blue at pH 2.5, colloidal iron, and metachromatic of tumor cells is feathery and foamy. Cytoplasmic
dyes. They resist hyaluronidase digestion. Acid vacuolization (indicative of mucin production)
mucin (simple, mesenchymal) contains hyaluronic can be seen.
acid and does not stain with PAS, but does stain with
alcian blue at pH 2.5, colloidal iron, and metachro- A-70. (c) Abundant cytoplasm
matic dyes. They digest with hyaluronic acid and In small cell carcinomas, tumor cells are relatively
can be found in sarcomas and mesotheliomas. Acid small, and they are two- to threefold in size of mature
mucin (complex, or sulfated, epithelial) is found in lymphocytes; however, in small cell carcinoma large
adenocarcinomas; PAS is usually positive; alcian cell variant, the size of tumor cells can be much
blue is positive at pH 1, and colloidal iron, mucicar- larger. Tumor cells show scant cytoplasm and high
mine, and metachromatic stains are also positive. N/C ratio, large nuclei with fine chromatin (salt-and-
They resist the digestion with hyaluronidase. Finally, pepper) pattern, nuclear crowding and molding, and
acid mucin (complex, connective tissue) are found paranuclear blue bodies in the cytoplasm. The nucle-
in stromal tissue such as cartilage and bone, and oli are inconspicuous in small cell carcinomas.
they are PAS negative but do stain selectively with Abundant cytoplasm is not a feature seen in small
alcian blue at pH 0.5. cell carcinoma. It can be seen in carcinoid and large
cell neuroendocrine tumor.
A-68. (d) Mucicarmine
There are a variety of mucin stains. In the colloidal A-71. (a) Fine granular (salt-and-pepper) chromatin
iron stain, iron particles are stabilized in ammonia pattern
and glycerin and are attracted to acid mucopolysac- The World Health Organization (WHO) classifica-
charides. It requires formalin fixation. Phospholipids tion recognizes four major types of lung neuroendo-
and free nucleic acids may also stain. The actual crine tumors: typical carcinoid, atypical carcinoid,
blue color comes from a Prussian blue reaction. The small cell lung cancer (SCLC), and large cell neuro-
tissue can be predigested with hyaluronidase to pro- endocrine carcinoma (LCNEC). The classification is
vide more specificity. In alcian blue stain, the pH of based on morphology, mitotic rate, and the absence
the stain can be adjusted to improve the specificity. or presence of necrosis. Ki-67 (MIB1) labeling is
In PAS stain, both glycogen and mucins are stained, also play an important role in the diagnosis and clas-
but the tissue can be predigested with diastase to sification of these tumors. In histological section, the
remove glycogen. In mucicarmine stain, epithelial characteristics of LCNEC include three features:
mucins are stained. It is the most specific mucin high-grade nuclei, neuroendocrine morphology, and
stain for adenocarcinoma, but its sensitivity is low. positive neuroendocrine IHC markers. In cytological
The most sensitive mucin stain is PAS, but its speci- preparations, LCNEC typically reveals as dispersed
ficity is poor. Colloidal iron stains are unpredictable. individual cells or loosely formed clusters. The size
Alcian blue stains are simple, but have a lot of back- of cells is usually larger than those of SCLC. Tumor
ground staining. cell has prominent nucleoli, fine rather than coarse
granular chromatin (seen in adenocarcinoma), and
A-69. (d) Poorly differentiated adenocarcinoma more markedly cytological pleomorphism, whereas
The differential diagnosis of spindle cells in fluid the reactivities for synaptophysin and chromogranin
cytology is broad, including spindle cell carcinoma, may be weak in small cell lung carcinoma. Tumor
sarcomatoid mesothelioma, melanoma, and all other cells have coarse granular rather than fine chromatin
tumors with spindle cell morphology. Therefore, and lacy or vacuolated cytoplasm. In difficult cases,
careful review of the patient’s malignant history is diagnosis of LCNEC versus a poorly differentiated
extremely important, and it may help to narrow adenocarcinoma depends on IHC staining of
192 Q.K. Li and W.E. Khalbuss

neuroendocrine markers. Although LCNEC are posi- (eccentric nuclei with abundant cytoplasm, some-
tive for neuroendocrine markers, 10–20 % of non- times containing a perinuclear clearing) to anaplastic
small cell lung cancers label for neuroendocrine (very large round or polygonal cells with bizarre,
markers. Therefore, the differential diagnosis of pleomorphic nuclei). Anaplastic forms may also con-
these two tumors should be based on the combina- tain multinucleated and/or Reed–Sternberg-like cells.
tion of cytomorphology and IHC study. Immunophenotypically, PEL cells reveal a “null”
lymphocyte phenotype (i.e., CD45 is expressed), but
A-72. (b) Make cell block as usual routine B-cell (including surface and cytoplasmic
Making a cell block preparation is important in fluid immunoglobulin, CD19, CD20, CD79a) and T-cell
cytology, particularly in cases with atypical epithe- (CD3, CD4, CD8) markers are negative. Instead, vari-
lial cells and/or suspicious metastatic carcinomas. In ous markers of lymphocyte activation (CD30, CD38,
such cases, IHC study is usually helpful in the dif- CD71, human leukocyte antigen DR) and plasma
ferential diagnosis. Repeat tap and biopsy of the lung cell differentiation (CD138) are usually displayed.
lesion are not necessary. The main differential diag- Molecular studies have provided evidence that
nosis is reactive mesothelial cells versus adenocarci- immunoglobulin gene rearrangements and somatic
noma. Reactive mesothelial cells show a spectrum of mutation have occurred in PEL cells. Cytogenetic
changes ranging from normal to markedly atypical analysis has revealed complex karyotypes but no
cells. In adenocarcinomas, tumor cells form three- common chromosomal abnormality in PEL.
dimensional clusters with smooth edges. They have
hyperchromatic nuclei, irregular nuclear membrane, A-74. (d) Hepatocellular carcinoma
prominent nucleoli, scant cytoplasm, and without “Intranuclear inclusions” have been described in a
intercellular “windows.” In addition, immunostains variety of malignant tumors, such as papillary thy-
of mesothelial cell marker (calretinin, D2-40) and roid carcinoma, lung adenocarcinoma, meningioma,
adenocarcinoma markers (TTF, Napsin A, BerEP4, hepatocellular carcinoma, and others. It has not been
B72.3, CEA, CD15) may help in the differential found in renal cell carcinomas. The cytological
diagnosis. features of renal cell carcinomas (clear cell renal
cell carcinomas) include loosely cohesive clusters
A-73. (d) All of the above and/or scattered individual tumor cells. They have
Primary effusion lymphoma (PEL) is a rare HIV- slightly enlarged round nuclei with or without
associated non-Hodgkin lymphoma (NHL) that nucleoli depending on the Fuhrman grade of the
accounts for approximately 4 % of all HIV- tumor. The background of smears can be bloody
associated NHL. PEL has a unique clinical predilec- with vascular structures, best seen on cell block
tion for arising in body cavities such as the pleural preparations. Tumor cells also have abundant vesic-
space, pericardium, and peritoneum. There is an evi- ular cytoplasm and cytoplasmic vacuoles. The N/C
dence of human herpesvirus (HHV)-8 infection in ratio is usually in normal range.
patients. The exact oncogenic mechanisms of
HHV-8 have not been clearly defined. Treatment is A-75. (c) Prepare cell blocks and make sure that it have
usually with combination CHOP (cyclophospha- adequate tumor cells
mide, doxorubicin, vincristine, and prednisone) che- Currently EGFR, KRAS, and other mutations are
motherapy and antiretroviral therapy (if HIV routinely tested in non-small cell lung cancer
positive). The prognosis for PEL is poor, with a patients. Any type of formalin-fixed and paraffin-
median survival time of around 6 months. The diag- embedded specimen, including cell block prepara-
nosis of PEL requires cytomorphological analysis of tions, can be used for these molecular and mutational
effusion specimen, immunophenotypic and molecu- analyses. For any type of specimens, the presence of
lar tests, and virologic criteria (i.e., previously infec- adequate tumor cells is crucial and necessary to be
tion with HHV-8). The diagnosis is usually made on confirmed prior to sending the specimen for molecu-
a cytological preparation (e.g., liquid-based prepara- lar tests. In lung adenocarcinoma, the EGFR and
tion, cytospin, cell block) of the effusion fluid. KRAS mutational rates are 10–15 and 15–20 % in
Morphologically, tumor cells are large and have the United States. EGFR mutation is associated with a
round to irregular nuclei, prominent nucleoli, and good response to tyrosine kinase inhibitors therapy,
varying amounts of cytoplasm. The cells show a range whereas KRAS mutation is associated with a resis-
of appearances, from immunoblastic (round nuclei tance to tyrosine kinase inhibitor therapy in lung
with central prominent nucleoli) to plasmablastic cancer patients. It is not necessary to obtain tissue
2 Serous Fluid Cytopathology 193

from primary tumor; molecular studies can be negative for HHV-8. In PEL, tumor cells are negative
performed using tissue from a metastatic tumor. for Epstein–Barr virus, but positive for HHV-8.

A-76. (d) All of the above A-79. (c) Colon


Psammoma body can be found in both benign and A metastatic colonic adenocarcinoma has these
malignant conditions. In benign conditions such as classic cytological features. Tumor cells showed tall
mesothelial cell hyperplasia and endosalpingiosis, the columnar “picket fence” appearance, hyperchromatic
presence of psammoma body is a common finding. pencil-shaped nuclei, coarse chromatin, and promi-
In malignant conditions such as ovarian serous car- nent nucleoli. The presence of “dirty” necrosis is usu-
cinoma and mesothelioma, numerous psammoma ally not as apparent as seen in histological sections.
bodies can be identified in ascites specimens. In breast carcinomas, tumor cells form tight three-
dimensional “cannon ball” clusters. The size of tumor
A-77. (c) Pyknotic nuclei cells is smaller than that of colon adenocarcinomas;
In a fluid specimen, tumor cells of metastatic squa- they may have intranuclear inclusion. Renal cell car-
mous cell carcinoma are usually rounded up and cinomas usually reveal a centrally located nuclei and
form loosely clusters. The cytological features of clear cytoplasm. Lung adenocarcinomas may have a
tumor cells include large nuclei with smudgy chro- variety of morphological features. The finding of tall
matin, nuclei with variation in size and shape, and columnar cells favors a colonic primary rather than
dense cytoplasm (indicative of cytokeratin forma- the lung primary; IHC stain of TTF, Napsin A, CK7,
tion), except the feature of pyknotic nuclei. Pyknotic CK20, and CDX2 may aid to the differential diagnosis
nuclei are usually seen reactive and/or benign squa- in difficult cases.
mous cells. Finally, prominent nucleoli can be seen
in poorly differentiated squamous cell carcinomas A-80. (a) Prominent nucleoli
and should not be confused with poorly differenti- The presence of prominent nucleoli is not the feature
ated adenocarcinomas. Tumor cells of adenocarci- seen in small cell carcinomas. All other features can be
noma have vesicular nuclei, coarse chromatin, and seen in both adenocarcinoma and small cell carci-
vacuolated cytoplasm. noma. In adenocarcinomas, tumor cells form acini
and/or three-dimensional clusters and reveal hyper-
A-78. (a) Burkitt’s or Burkitt’s-like lymphoma (BL) chromatic nuclei, vesicular or coarse chromatin,
The most common histological types of AIDS- prominent nucleoli, and vacuolated cytoplasm (indic-
related non-Hodgkin lymphoma (ARL) include ative of mucin production). All neuroendocrine tumors
Burkitt’s or Burkitt’s-like lymphoma (BL), diffuse share many cytological features, such as fine granular
large B-cell lymphoma (DLBCL) (including the (salt-and-pepper) chromatin and acini/rosette arrange-
subtypes immunoblastic (including most cases of ments of tumor cells. In addition, in small cell carci-
PCNSL) and centroblastic lymphoma), plasmablas- nomas, nuclear crowding and molding are common
tic lymphoma (including oral cavity involved and features. Tumor necrosis can be seen in both adeno-
multicentric Castleman’s disease [MCD] associ- carcinoma and small cells carcinoma.
ated), and primary effusion lymphoma (PEL).
Among them, BL rarely presents as a lymphoma- A-81. (b) Making cell block to perform IHC markers
tous effusion. BL tumor cells are positive for the S100, HMB45, and melanin A
characteristic c-myc gene rearrangement and show Metastatic melanoma to serosal cavities, including
no evidence of HHV-8 infection. Peripheral T-cell pleural, pericardial, and peritoneal cavities, is not
NHLs presenting with an effusion can usually be uncommon. Tumor cells are usually rounded up in a
distinguished by routine immunohistochemistry for fluid specimen and may or may not produce melanin
T-cell markers or T-cell receptor (TCR) gene rear- pigment, particularly in desmoplastic/spindle cell
rangement studies. Because of similar morphology, melanoma (no pigment production). The cytological
T-cell anaplastic large cell lymphoma may also be features of a metastatic melanoma include individ-
confused for PEL in some cases; immunohisto- ual large cells with prominent nucleoli, intranuclear
chemical staining for anaplastic lymphoma kinase inclusion, cytoplasmic pigment, binucleation, and
in addition to the TCR gene rearrangement studies the presence of plasmacytoid cells. In desmoplastic/
may be helpful in these cases. Both DLBCL and spindle cell melanoma, tumor cells reveal bizarre
PEL can have similar cytomorphology. In DLBCL, nuclei and prominent nucleoli. Immunomarkers
tumor cells are positive for Epstein–Barr virus, but S100, HMB45, and MelanA/Mart1 are helpful in the
194 Q.K. Li and W.E. Khalbuss

differential diagnosis. S100 stains nuclei and cyto- cytology, tumor cells of metastatic breast carcinoma
plasm with 90 % sensitivity but not specific. HMB45 form tight three-dimensional “cannon ball” clusters.
stains cytoplasm; it is more specific but less sensitive The size of tumor cells is relatively smaller than that
than S100; only 20 % of desmoplastic/spindle cell of metastatic adenocarcinoma from the lung and/or
melanomas are immunoreactive with HMB45. GI tract. The presence of intracytoplasmic lumina
MelanA/Mart1 stains cytoplasm, and is a sensi- (magenta body) is also one of the characteristics of
tive marker, but it also stains steroid-producing metastatic breast carcinoma.
cells in the ovary, testis, and adrenal cortex; it is
also negative in desmoplastic/spindle cell mela- A-85. (a) Hyperchromatic nuclei and prominent nucleoli
noma. Although unstained cytospin slides can be The presence of prominent nucleoli is not the feature
used for IHC study, cell block preparation provides seen in small cell carcinomas. All other features can
better results. The combination of S100, HMB45 be seen in small cell carcinoma. In small cell carcino-
and melanin A may result in the best immunopattern mas, tumor cells form three-dimensional clusters or
in the differential diagnosis of melanomas. dispersed individual cells and reveal hyperchromatic
nuclei, fine granular (salt-and-pepper) chromatin,
A-82. (d) Naked nuclei and scant cytoplasm. In addition, nuclear crowding,
The presence of naked nuclei is not the feature seen molding, apoptotic body, and mitotic figures are com-
in metastatic breast carcinoma. In pleural effusion mon features. The background of the smear may also
cytology, tumor cells of metastatic breast carcinoma reveal “blue strips” (indicative of breakdown nuclear
form tight three-dimensional “cannon ball” clusters. material). In adenocarcinomas, tumor cells form
The size of tumor cells is relatively smaller than that acini and/or three-dimensional clusters and reveal
of metastatic adenocarcinoma from the lung and/or hyperchromatic nuclei, vesicular or coarse chroma-
GI tract. The presence of intracytoplasmic lumina, tin, prominent nucleoli, and vacuolated cytoplasm
so-called magenta body, is also one of the character- (indicative of mucin production).
istics of metastatic breast carcinoma. Magenta
bodies are variably sized red-to-purple perinuclear A-86. (d) All of the above
inclusions seen by the Diff-Quik stains. The promi- Lymphocytic effusion is the effusion in which
nent nucleoli are a common cytological feature seen almost all nucleated cells are lymphocytes. In the
in a variety of adenocarcinomas. Naked nuclei can cytological preparation, it reveals small lympho-
be seen in metastatic melanoma and hepatocellular cytes. In tuberculosis, the identification of eosino-
carcinoma. phils is important; it may be suggestive of the
diagnosis. The diagnosis of tuberculosis can be con-
A-83. (c) Hyaluronic acid–alcian blue firmed by microbiology culture or pleural biopsy.
Both mesothelioma and adenocarcinoma produce Malignancy is the common cause of lymphocytic
mucins. In adenocarcinomas, tumor cells are positive effusion. It may be caused by the obstruction of lym-
for mucicarmine, PAS, and cytokeratin. Similarly, in phatic outflow by the tumor or peritumoral lympho-
mesotheliomas, particularly in epithelial mesothelio- cytic reaction. Repeat tap and cytological evaluation
mas, tumor cells may also be positive for mucicar- may increase the chance of finding tumor cells.
mine, PAS, and cytokeratin. Studies have shown that Small lymphocytic lymphoma/chronic lymphocytic
mucicarmine and PAS stains were usually eradicated leukemia may cause lymphocytic effusion, particu-
or reduced in intensity by pretreatment of the tissue larly in a patient with peripheral lymphocytosis.
sections with hyaluronidase, suggesting that hyal- Flow cytometry and immunostain of B-cell markers
uronic acid was responsible for the positive mucin may help in the diagnosis. Other causes, such as
reactions in mesothelioma. Therefore, hyaluronic status for post coronary artery bypass may also
acid–alcian blue stain is a characteristic feature of a induce a lymphocytic effusion.
mesothelioma rather than an adenocarcinoma. In
alcian blue stain, the pH of the stain can be adjusted A-87. (d) CD56
to improve the specificity. CD56 is considered to be the most sensitive neuro-
endocrine marker for small cell lung carcinoma,
A-84. (d) Bipolar nuclei whereas the reactivities for synaptophysin and
The presence of bipolar nuclei is not the feature seen chromogranin A are typically weak in small cell lung
in metastatic breast carcinoma. In pleural effusion carcinoma. Approximately 25 % of small cell lung
2 Serous Fluid Cytopathology 195

carcinomas are negative for both synaptophysin A-90. (d) Immunostains of synaptophysin, chromo-
and chromogranin A, but most of these tumors are granin, CD56, and CD45
positive for CD56. Still, 10 % of small cell lung An important differential diagnosis of small cell lung
carcinomas are negative for all three commonly carcinoma is lymphoma. In small cell carcinomas,
used neuroendocrine markers. The frequency of tumor cells form three-dimensional clusters or dis-
neuroendocrine marker negativity in small cell lung persed individual cells and reveal hyperchromatic
carcinoma is even higher in small biopsy and cytology nuclei, fine granular (salt-and-pepper) chromatin, and
specimens, in which focal reactivity may not be rep- scant cytoplasm. In addition, nuclear crowding, mold-
resented on the smear due to the nature of the speci- ing, apoptotic body, and mitotic figures are common
men and the heterogeneity of the tumor. If the features. The background of the smear may also reveal
cytomorphology is classic for small cell carcinomas, “blue strips” (indicative of breakdown of nuclear
the lack of supporting immunohistochemical stains material). Positive immunostains of synaptophysin,
should not serve as evidence against the diagnosis. chromogranin and CD56, and negative stain of CD45
TTF1 is positive for both small cell lung carcinoma may confirm the diagnosis. In lymphomas, tumor cells
and lung adenocarcinomas. are discohesive and dispersed throughout the smear.
The finding of a “lymphoglandular body” is charac-
A-88. (b) False teristic of lymphoma. The chromatin of lymphoma
TTF1 (thyroid transcription factor 1) is expressed cells are coarse granular, and they may have promi-
by thyroid and pulmonary cells and is positive in nent nucleoli. Flow cytometry and immunostain of
approximately 70–80 % of small cell lung carcino- lymphoma markers may help in the diagnosis.
mas. TTF1 is usually used to identify the pulmonary Immunomarkers of AE1/AE3, TTF, and BerEP4 are
(and/or thyroid) origin of metastatic adenocarcino- best used for carcinomas, particularly adenocarci-
mas. In contrast to adenocarcinomas, TTF1 in small noma. CD20 and CD5 may stain background lym-
cell carcinomas cannot be used to distinguish the phocytes in the smear of a small cell carcinoma.
origin of the tumor, i.e., pulmonary versus extrapul-
monary origin. In addition to lung small cell carci- A-91. (b) Irregular nuclear membrane and
nomas, the expression of TTF1 has been found to hyperchromasia
occur in 20–80 % of small cell carcinomas of vari- Reactive cellular changes can happen in a variety of
ous sites, including the prostate, bladder, cervix, and settings, such as radiation and chemotherapy. Reactive
gastrointestinal tract. mesothelial cell may form cohesive sheets and/or
two-dimensional clusters of cells. These cells can be
A-89. (b) Small cell lung carcinoma larger in size and have enlarged nuclei, but the normal
Both adenocarcinomas and small cell carcinomas are N/C ratio is maintained. Multinucleation and promi-
cytokeratin positive. The weak punctate (“dotlike”) nent nucleoli are common. Intercellular “windows”
labeling pattern for cytokeratins, including both AE1/ are a feature of mesothelial cells. However, hyper-
AE3 and CAM 5.2, is the feature commonly seen in chromatic nuclei, clumped chromatin, and irregular
small cell lung carcinomas. In contrast, a strong cir- nuclear membranes are not features seen in reactive
cumferential labeling of tumor cells is characteristic atypia. The presence of these features indicates
of a non-small cell lung carcinoma, particularly an malignant lesions. The differential diagnosis of
adenocarcinoma. However, this feature may be seen reactive mesothelial cells includes mesothelioma
in other high-grade carcinomas. Although all resected and carcinoma, particularly a well-differentiated
small cell carcinomas are cytokeratin positive, some adenocarcinoma. In adenocarcinomas, tumor cells
tumors may show weak and minimal reactivity. Thus, form three-dimensional clusters with smooth edges.
on small biopsy and/or cytology specimens, a subset They have hyperchromatic nuclei, irregular nuclear
of small cell carcinomas may appear cytokeratin neg- membranes, prominent nucleoli, scant cytoplasm,
ative. In the absence of confirmatory markers, it is and without intercellular “windows.”
important to exclude the possibility of lymphoma,
basaloid squamous cell carcinoma, and other small A-92. (b) CT-guided transthoracic needle biopsy
round blue cell tumors. If the cytomorphology is The most common primary malignant lesions
classic for small cell carcinomas, the lack of sup- involving the pleural cavity are mesotheliomas and
porting immunohistochemical stains should not lymphomas. Malignant pleural effusion is a condition
serve as evidence against the diagnosis. in which cancer causes an abnormal accumulation
196 Q.K. Li and W.E. Khalbuss

of fluid between the thin layers of pleura lining the sized green- or pink-red-colored clumps by the
outside of the lung and the wall of the chest cavity. Papanicolaou stain. It is believed that the amorphous
Lung cancer and breast cancer account for about material is different from fibrin. Diagnosis of rheu-
50–65 % of malignant pleural effusions. Among matoid pleuritis relies on the characteristic cytological
these procedures, blind transthoracic needle biopsy findings in an exudative pleural fluid.
has the sensitivity of less than 50 %; thoracentesis
cytology has a sensitivity of 60 %; ultrasound- A-96. (c) Mesothelial cells
guided transthoracic needle biopsy has a sensitivity The diagnosis features of a rheumatoid pleuritis
of 73 %; and a CT-guided transthoracic needle rely on the characteristic cytological findings of the
biopsy has a sensitivity of 87 %. exudative pleural fluid, which contains elongated
and giant multinucleated histiocytes in a sea of
A-93. (d) Ultrasound-guided thoracentesis amorphous granular material. The absence of meso-
The ultrasound-guided thoracentesis has a specific- thelial cells is also characteristic of the disease.
ity of 100 % for distinguishing malignant pleural
effusions from other causes of pleural effusion, A-97. (d) All of the above
based on the presence of visible pleural lesions (i.e., Charcot–Leyden crystal is a needle-shaped crystal.
metastases), pleural thickening greater than 1 cm, They vary in size and may be as large as 50 μm in
pleural nodularity, diaphragmatic thickening greater length. They consist of lysophospholipase, an
than 7 mm, and an echogenic swirling pattern visi- enzyme synthesized by eosinophils, and are pro-
ble in the pleural fluid. duced from the breakdown of these cells. It was first
described by Friedrich Albert von Zenker in 1851.
A-94. (b) False The finding of Charcot–Leyden crystals in pathol-
Cytological features of adenocarcinomas include ogy specimen is indicative of a disease involving
three-dimensional clusters, acinar and papillary eosinophilic inflammation or proliferation, such as
groups, or honeycomb-like sheets of tumor cells. in allergic reactions, parasitic infections, and eosin-
Tumor cells reveal large round- or oval-shaped ophilic effusions.
nuclei, coarse chromatin, irregular nuclear mem-
brane, and prominent nucleoli. The cytoplasm of A-98. (d) LE cells contain nuclear material with visible
tumor cells is feathery and foamy (indicative of chromatin structures, such as coarse chromatin
mucin production). Squamous cell carcinomas granules
account for 30–40 % of non-small cell lung cancers. The lupus pleuritis may occur in one third of lupus
In metastatic squamous cell carcinomas, the cyto- patients. The finding of LE cells in the fluid cyto-
logical features of tumor cells include large nuclei logical preparations is characteristic and can be
with smudgy chromatin, nuclei with variation in size identified in the majority of cases. The LE cell was
and shape, and dense cytoplasm (indicative of cyto- discovered in bone marrow by Hargraves et al. in
keratin formation). Tumor cells are usually rounded 1948. The LE cell is a neutrophil or macrophage that
up in the fluid specimens. The formation of two- and has phagocytized (engulfed) the denatured nuclear
three-dimensional tumor cell clusters is not uncom- material (hematoxylin body) of another cell.
mon, particularly in poorly differentiated squamous Denatured nuclear material in the neutrophil/mac-
cell carcinomas. In addition, prominent nucleoli rophage has a glassy and homogeneous appearance;
can be seen in poorly differentiated squamous cell it may push the nuclei to the peripheral location.
carcinomas and should not be confused with poorly The hematoxylin body may be green, blue, or pur-
differentiated adenocarcinomas. ple in color with the Papanicolaou stain and magenta
color with the Diff-Quik stains. Cells contain
A-95. (d) All of the above nuclear material with visible chromatin structures
The rheumatoid pleuritis may occur in approxi- (i.e., coarse chromatin pattern) are the so-called Tart
mately 2–3 % of patients with rheumatoid arthritis. cells; they are not LE cells.
The pleural effusion can be unilateral or bilateral.
Cytological features include abundant granular A-99. (d) All of the above
debris, multinucleated giant cells, and scattered In effusion as well as ascites and pericardial effu-
lymphocytes. The presence of granular debris is sion, the presence of a second population of cells,
characteristic of the rheumatoid pleuritis. Granular numerous large clusters of cells, and lacunae spaces on
debris are amorphous material and form variable cell block preparation is characteristic of a malignant
2 Serous Fluid Cytopathology 197

effusion. The cellularity in a malignant effusion may Ellis CL, Burroughs F, Michael CW, Li QK. Cytology of metastatic
be markedly increased. Numerous tumor cell clus- renal medullary carcinoma in pleural effusion. A study of two cases.
Diagn Cytopathol. 2009;37:843–8.
ters are seen in the specimen. In addition to mesothe- Gupta S, Sodhani P, Jain S. Cytomorphological profile of neoplastic
lial cells, metastatic tumor cells represent a second effusions: an audit of 10 years with emphasis on uncommonly
cellular component in the fluid specimen. Lacuna is encountered malignancies. J Cancer Res Ther. 2012;8:602–9.
the empty space surrounding the tumor cell cluster on Henderson DW, Reid G, Kao SC, van Zandwijk N, Klebe S. Challenges
and controversies in the diagnosis of mesothelioma: Part 1.
the cell block preparation. It has been reported that Cytology-only diagnosis, biopsies, immunohistochemistry, discrim-
lacunae have been identified in 75 % of malignant ination between mesothelioma and reactive mesothelial hyperpla-
effusions, particularly in adenocarcinomas. sia, and biomarkers. J Clin Pathol. 2013;66(10):847–53.
Jing X, Li QK, Bedrossian U, Michael CW. Morphologic and immun-
mocytochemical performances of effusion cell blocks prepared
A-100. (b) False using three different methods. Am J Clin Pathol. 2013;139:177–82.
Lacuna is the empty space surrounding the tumor Khalbuss WE, Yang H, Lian Q, Elhosseiny A, Pantanowitz L, Monaco
cell cluster on the cell block preparation. It is an arti- SE. The cytomorphologic spectrum of small-cell carcinoma and
fact and has been found in 75 % of malignant effu- large-cell neuroendocrine carcinoma in body cavity effusions: a
study of 68 cases. Cytojournal. 2011;8:18.
sions. The finding of lacunae at low magnification is Khalbuss WE, Monaco SE, Pantanowitz L. Chapter 9. Serous effusions.
helpful for identifying suspicious cells. Although it In: The ASCP quick compendium (QC) of cytopathology. Chicago:
is a common finding in a malignant effusion, lacu- ASCP Press; 2013. p. 128–59.
nae can also be seen in benign conditions. Therefore, Li Q, Bavikatty N, Michael CW. The role of immunohistochemistry in the
distinguishing squamous cell carcinoma from mesothelioma and ade-
evaluation of cells at high magnification is necessary nocarcinoma in pleural effusion. Semin Diagn Pathol. 2006;23:15–9.
to confirm the diagnosis of a malignant effusion. Longatto-Filho A, Bisi H, Bortolan J, Granja NV, Lombardo
V. Cytologic diagnosis of metastatic sarcoma in effusions. Acta
Cytol. 2003;47:317–8.
Nauen D, Li QK. Cytological diagnosis of metastatic glioblastoma in
Reading List the pleural effusion of a lung transplant patient. Diagn Cytopathol.
2014;42(7):619–23.
Abadi MA, Zakowski MF. Cytologic features of sarcoma in fluids. Stoll LM, Johnson MW, Gabrielson E, Askin F, Clark DP, Li QK. The
Cancer Cytopathol. 1998;84:71–6. utility of Napsin-A in the identification of primary and metastatic
Bibbo M, Wood MD, Fitzpatrick BT. Peritoneal washings and ovary. In: lung adenocarcinoma among cytologically “poorly differentiated
Bibbo M, Wilbur D, editors. Comprehensive cytopathology. 3rd ed. carcinoma”. Cancer Cytopathol. 2010;118:441–9.
Philadelphia: Saunders/Elsevier; 2008. Tong LC, Ko HM, Saieg MA, Boerner S, Geddie WR, da Cunha SG.
Cagle PT. Pleural histology. In: Light RW, Lee YCG, editors. Pleural Subclassification of lymphoproliferative disorders in serous effu-
disease: an international textbook. London, England: Arnold sions: a 10-year experience. Cancer Cytopathol. 2013;121:261–70.
Publishers; 2003. p. 249–55. Wheeler YR, Burroughs F, Li QK. Fine needle aspiration of a well-
Cibas ES. Peritoneal washings. In: Cibas ES, Ducatman BS, editors. differentiated papillary mesothelioma in the hernia sac: case report
Cytology: diagnostic principles and clinical correlates. Philadelphia: and review of literature. Diagn Cytopathol. 2009;37:748–54.
Saunders/Elsevier; 2009. Wojcik EM, Naylor B. “Collagen balls” in peritoneal washings.
DeMay RM. The art and science of cytopathology, exfoliative cytology, Prevalence, morphology, origin and significance. Acta Cytol.
vol. 1. 2nd ed. Chicago: ASCP Press; 2012. 1992;36(4):466–70.
http://www.springer.com/978-1-4939-1476-0

You might also like